Sunteți pe pagina 1din 68

Din sumar :

* Articole ]i note matematice


* Concursuri ]i olimpiade  pentru clasele V - XII
* 100 de probleme propuse  pentru clasele I - IV
* 175 de probleme teme  pentru clasele V - XI
* Lucr[ri de verificare  pentru clasele V - XII
* Probleme propuse  pentru clasele V - XII
* Rubrica rezolvitorilor

Anul XXV , Nr. 2


2014 / 2015

Publicatie semestriala pentru elevi


)

,
CLASELE I-XII
R.M.C.

Sumar
1. ARTICOLE }I NOTE MATEMATICE
DOU{ #NCADR{RI }I O RAFINARE A INEGALIT{|II MEDIILOR CLASICE - prof. dr. Dorin
M[rghidanu, prof. Leonard Giugiuc . . . . . . . . . . . . . . . . . . . . . . . . . . . . . . . . . . . . . . . . 1
 AN APPLICATION OF STIRLING’S FORMULA - prof. Shafiqur Rahman, prof. Leonard

Giugiuc . . . . . . . . . . . . . . . . . . . . . . . . . . . . . . . . . . . . . . . . . . . . . . . . . . . . . . . . . . . . 5
 EXISTEN|A }I CONSTRUC|IA UNOR TRIUNGHIURI CLASICE - prof. Marian Dinc[ . ..... 7
 ASUPRA UNEI INEGALIT{|I - prof. Marian Cucoane] . . . . . . . . . . . . . . . . . . . . . . . . . . . 9
2. CONCURSURI }I OLIMPIADE: FAZA LOCAL{, DOLJ  Clasele V - XII . . . . . . . . . . 10
3. PROBLEME PROPUSE  CICLUL PRIMAR . . . . . . . . . . . . . . . . . . . . . . . . . . . . . . . 15
4. PROBLEME - TEME
 Gimnaziu . . . . . . . . . . . . . . . . . . . . . . . . . . . . . . . . . . . . . . . . . . . . . . . . . . . . . . . . . . 23

 Liceu . . . . . . . . . . . . . . . . . . . . . . . . . . . . . . . . . . . . . . . . . . . . . . . . . . . . . . . . . . . . . 32

5. LUCR{RI DE VERIFICARE  Clasele V - XII . . . . . . . . . . . . . . . . . . . . . . . . . . . . . . . 41


6. PROBLEME PROPUSE
 Gimnaziu . . . . . . . . . . . . . . . . . . . . . . . . . . . . . . . . . . . . . . . . . . . . . . . . . . . . . . . . . . 58

 Liceu . . . . . . . . . . . . . . . . . . . . . . . . . . . . . . . . . . . . . . . . . . . . . . . . . . . . . . . . . . . . . 61

7. RUBRICA REZOLVITORILOR . . . . . . . . . . . . . . . . . . . . . . . . . . . . . . . . . . . . . . . . . . 64

Redac\ia R.M.C.
Director editor : prof. D[nu\ Dr[cea
Redactor coordonator : prof. Dan Secl[man
Colectivul de redac\ie : @nv. Mariana Ungureanu, prof. Constantin Basarab, prof. Constantin
Cazacu, prof. Dorina Dr[cea,prof. Liliana Niculescu, prof. Ion P[tra]cu,
prof. Daniel Al. Ion, prof. Nicolae T[l[u, prof. Lucian Tu\escu,
prof. dr. Grigorie Preoteasa, prof. Ioan Stana, prof. Felician Preda,
prof. Octavian Ungureanu, prof. C[t[lin Cristea, prof. Raluca Ciurcea,
prof. Cristian Paul Moan\[, prof. Cerasela Bociu, prof. Delia Mitran.
Colaboratori : prof. Dan Popescu (Suceava), prof. Traian Ianculescu (Zimnicea), prof. Emilia Velcea
(Lupeni), prof. univ. dr. Dumitru Acu (Sibiu), prof. univ. dr. Cristinel Mortici
(T`rgovi]te), prof. univ. dr. Florentin Smarandache (New Mexico, Arizona, U.S.A),
prof. dr. Dorin M[rghidanu (Corabia).
Adresa redac\iei: EDITURA CARDINAL:
Cartier George Enescu, Str. R[zboieni, Nr. 2, Bl. B13, Sc. 1, Ap. 2, Craiova, Cod po]tal: 200667
Telefon: 0744.555376; 0351.422424 Fax: 0351.422425
© Editura CARDINAL
 Tehnoredactare computerizat[:
E

All Rights Reserved C Irina Liliana Suciu (Editura Cardinal)


REVISTA DE MATEMATIC{ DIN CRAIOVA 

Publica\ie semestrial[ pentru elevi ]i profesori

 E

C
ANUL XXV Nr. 2
anul ]colar 2014/2015

* ARTICOLE }I NOTE MATEMATICE *


***************************************************************************

 1. DOU{ #NCADR{RI }I O RAFINARE


A INEGALIT{|II MEDIILOR CLASICE

Prof. dr. DORIN M{RGHIDANU, Corabia, Olt


Prof. LEONARD GIUGIUC, Dr. Tr. Severin

#n nota de fa\[ sunt prezentate ]i demonstrate dou[ inegalit[\i de @ncadrare cu medii


clasice. Sunt ob\inute dou[ tipuri noi de medii care realizeaz[ o rafinare discret[ a acestora.
Pentru numerele reale pozitive a 1 , a 2 , . . ., a n , sunt cunoscute urm[toarele medii
n
clasice: A n a :  1
n   a k (media aritmetic[ a numerelor a 1 , a 2 , . . ., a n ), (1);
k1
n
G n a :  n  a k (media geometric[ a numerelor a 1 , a 2 , . . ., a n ), (2);
k1

H n a :  n (media armonic[ a numerelor a 1 , a 2 , . . ., a n ), (3).


n
 1
k1 a k
De asemenea cunoa]tem ]i inegalitatea dintre ele, A n a  G n a  H n a, (4).
(unde egalitatea are loc dac[ ]i numai dac[ a 1  a 2  . . .  a n ).
Pentru inegalitatea mediilor exist[ o literatur[ matematic[ impresionant[ (din care
selectiv ]i subiectiv prezent[m o bibliografie: pentru medii ]i demonstrarea inegalit[\ii
mediilor: 1  3, 6  14, 16, 17 ; pentru rafin[ri ale inegalit[\ii mediilor:
2, 3, 7, 11 ).
#n cele ce urmeaz[ vom realiza o rafinare a dublei inegalit[\i din (4), adic[ vom
intercala termeni noi @ntre elementele deja existente.
Ne vor interesa ]i vom studia @n cele ce urmeaz[ ]i urm[toarele dou[ sume cu
n n ak
radicali: S n a :   1 , (5). T n a :   , (6).
k1 1  ak k1 1  ak
Despre S n a vom demonstra o inegalitate - foarte interesant[ ]i @n sine (]i care a
avut drept enun\[ri preliminare - c`teva post[ri pe site-uri de matematic[ online, vezi
de exemplu 4, 5, 15).
1. Lem[ (@ncadrare cu medii pentru suma S n )
Fie n  2 un num[r natural. Pentru numerele reale a 1 , a 2 , . . ., a n  (0, 2 ) are loc
n
dubla inegalitate, n  1  n , (7).
1  A n a k1 1  a k 1  G n a
Demonstra\ie
Pentru inegalitatea din st`nga, consider[m func\ia:
f : (0,  )  R, f (x )  1 1
 (1  x ) 2 .
1x
Cum avem, f x   (1  x )  0, f  (x )  3 (1  x ) 2  0, rezult[ c[ f este
1
( )  32 5
2 4
func\ie convex[ pe (0,  ).
Prin aplicarea inegalit[\ii lui Jensen pe intervalul (0,  ), avem:
1  n f (a )  f 1 n a  1  n 1  1 
n
1 
n k1 k n k1 k n k1 1  ak 1 n k1 1  ak
1  n  ak
k1
 n . Egalitatea are loc dac[ ]i numai dac[: a 1  a 2  . . .  a n .
1  An
Observa\ie. Prima inegalitate din (7) are loc pe @ntregul interval (0,  ).
Pentru inegalitatea din dreapta, cu substitu\iile b k  ln a k , ( )k  1, n, avem:
n n
a k  (0, 2 )  b k  ( , ln 2 ), iar  1  1 .
k1 1  ak k1 1  e bk
Vom considera func\ia  : ( , ln 2 )  R, (x )  1 , pentru care avem:
1  ex
  (x )   1  (1  e x )  e x ,   (x )  1  (1  e x )  e x  (e x  2 ).
3/2 5/2
2 4
Rezult[ c[  este func\ia concav[ pe ( , ln 2 ).
Alic`nd inegalitatea lui Jensen pentru func\ii concave, avem:
1  n (b )   1  n b  1  n 1  1 
n k1 k n k1 k n k1 1  e bk n
b k
1  e k1
1
n

2 Revista de matematic[ din Craiova


n n
 1  n  1  n 
k1 1  e bk b k1
n k1 1  ak n
 ln ak
1
1  e k1 1  e k1
n n

n n
 1  n  1  n 
k1 1  ak 1
 ak
n k1 1  ak 1  e ln Gn
1  e ln k1
n

n
 1  n .
k1 1  ak 1  Gn
Egalitatea are loc dac[ ]i numai dac[ a 1  a 2  . . .  a n .
Inegalitatea din Lem[ ne permite ]i ob\inerea unei @ncadr[ri pentru cealalt[ sum[,
dar prin intermediul unui alt cuplu de medii, anume (H n , G n ).
2. Corolar (incadrarea cu medii pentru suma T n )
Pentru n  2 un num[r natural ]i pentru numerele reale x 1 , x 2 , . . ., x n  (1/2,  ) are
H n x n xn G n x
loc dubla inegalitate, n   n , (8).
1  H n x k1 1  x k 1  G n x
Egalitatea are loc dac[ ]i numai dac[ x 1  x 2  . . .  x n .
Demonstra\ie Pentru numerele reale a 1 , a 2 , . . ., a n  (0, 2 ) din Lem[, vom nota
x k  a1k , ()k  1, n.
1 xk
Rezult[ c[ x k  1 ,  , iar  , ( )k  1, n. Totodat[ avem,
2 1  ak 1  xk
n n n n
A n a  1
n k1  x1k  H 1x , G n a  n k1
 a k  1n k1  x1k  G 1x .
 a k  n k1
n n
Inlocuind aceste exprim[ri @n inegalitatea (7), se ob\ine inegalitatea din enun\.
Inegalitatea din Lem[ permite ]i o rafinare a inegalit[\ii mediilor. Mai exact are loc
urm[toarea,
3. Propozi\ie (rafinare a inegalit[\ii dintre A n x ]i G n x)
Pentru n  2, num[r natural ]i pentru numerele reale a 1 , a 2 , . . ., a n  (0, 2 ) are loc
n 2  S 2n a
rafinarea, G n a   A n a, (9) egalitatea are loc dac[ a 1  a 2  . . .  a n .
S 2n a
Demonstra\ie Dubla inegalitate din (7) se poate scrie @n urm[toarele echivalen\e:
n  S n a  n  n2  S 2n a  n2 
1  A n a 1  G n a 1  A n a 1  G n a
1  G n a 1  A n a 2
  21   G n a  2n  1  A n a 
n 2 S n a n 2 S n a
n 2  S 2n a
 G n a   A n a.
S 2n a
Asem[n[tor avem o rafinare pentru cealalt[ pereche de medii, dar prin intermediul
sumei T n .

Editura Cardinal 3
4. Propozi\ie (rafinarea inegalit[\ii dintre H n x ]i G n x)
Pentru n  2, num[r natural ]i pentru numerele reale x 1 , x 2 , . . ., x n  (1/2,  ) are loc
rafinarea,
T 2 x
H n x  2 n 2  G n x, (10)
n  T n x
iar egalitatea are loc dac[ x 1  x 2  . . .  x n .
Demonstra\ia rezult[ din rela\ia (8), sau din propozi\ia anterioar[, prin substitu\ia
a k  x1k , k  1, n ]i @n acord cu nota\iile (1) - (3), (5), (6).
#n accep\iunea cea mai general[, media unei mul\imi de numere este un num[r
cuprins @ntre minimul ]i maximul mul\imii de numere respective. Mai exact pentru
numerele x1 , x2 , . . ., xn  I R, vom numi medie a lor, func\ia M : I n  I, pentru care
avem,
minx 1 , x 2 , . . ., x n   M(x 1 , x 2 , . . ., x n )  maxx 1 , x 2 , . . ., x n , (11).
Putem atunci enun\a:
5. Corolar
Pentru n  2, num[r natural ]i pentru numerele reale a 1 , a 2 , . . ., a n  1 , 2 ]i cu
2
nota\iile,
n 2  S 2n a T 2 a
M n a  , M n a  2 n 2 , au loc afirma\iile:
S n a
2 n  T n a
(a) H n a  M n a  G n a  M n a  A n a, (12);
cu egalitate dac[ a 1  a 2  . . .  a n .
(b) M n a, M n a sunt medii ale numerelor a 1 , a 2 , . . ., a n .
Demonstra\ie Afirma\ia (a) rezult[ din Propozi\iile 3 ]i 4, pe domeniul comun
(0, 2 )  1 ,   1 , 2 , iar afirma\ia (b) rezult[ din (a), deoarece H n a ]i A n a sunt
2 2
medii.

Bibliografie
1. Beckenbach E. F. & Bellman R., “Inequalities”, Springer-Verlag, Berlin-Heidelberg-
New-York, 1961.
2. Bullen P. S. & Mitrinovie D. S. & Vasie P. M., “Means and Their Inequalities”, D.
Reidel Publidshing Company, Dordrecht/Boston, 1988.
3. Bullen P. S., “Handbook of Means and Their Inequalities”, Kluwer Academic
Publishers, Dordrecht/Boston/London, 2003.
4. Giugiuc Leonard, http://www.facebook.com/photo.php?fbid=529269517210488&set=gm.
555767914559654&type=1&theater.
5. Giugiuc L., http://www.artofproblemsolving.com/Forum/viewtopic.php?f=52&t=618646.
6. M[rghidanu D., “Inegalitatea lui Lagrage este echivalent[ cu inegalitatea mediilor”,
@n “ARHIMEDE”, nr. 3-4, pp. 17-19, 2005.

4 Revista de matematic[ din Craiova


7. M[rghidanu D., Bencze M., “New Means and Refinements for AM-GM-HM
Inequalities” @n “OCTOGON Mathematical Magazine”, Vol. 13, No. 2, pp. 999-1001,
October, 2005.
8. M[rghidanu D., Bencze M., “A new Proof for AM-GM Inequalitiy”, in “OCTOGON
Mathematical Magazine”, Vol. 13, No. 2, pp.1021-1026, October, 2005.
9. M[rghidanu D., “O demonstra\ie a inegalit[\ii mediilor ( pornind de la o problem[ din
Crux Mathematicorum)”, @n “REVISTA DE MATEMATIC{ DIN TIMI}OARA”, anul XI,
pp. 6-7, nr. 1/2006.
10. M[rghidanu D., “O metod[ a lui Liouville de demonstrare a inegalit[\ilor” ,
“GAZETA MATEMATIC{”, seria A, Anul XXV (CIV), pp. 17-23, nr. 1/2007.
11. M[rghidanu D., “O demonstra\ie simultan[ pentru inegalitatea direct[ ]i invers[ a
mediilor”, @n “CREA|II MATEMATICE”, seria A, Anul II, pp. 11-14, nr. 1, 2007.
12. M[rghidanu D., Dou[ demonstra\ii scurte pentru inegalitatea mediilor”, @n
“CREA|II MATEMATICE”, seria B, Anul II, pp. 20-21, nr. 2/2007.
13. M[rghidanu D., “O inegalitate din analiz[ pentru dou[ demonstra\ii ale inegalit[\ii
mediilor”, @n revista “MINUS”, pp. 29-30, nr. 1/2008.
14. M[rghidanu D., Marinescu D. S., Cornea V., “O inegalitate echivalent[ cu
inegalitatea mediilor”, @n “REVISTA DE MATEMATIC{ DIN TIMI}OARA”, anul XIII
(Seria a IV-a), pp. 3-6, nr. 3/2008.
15. M[rghidanu D., Giugiuc L., https://www.facebook.com/photo.php?fbid=5304780537
56301&set=gm.1511379709150085&type=1&theater.
16. Mitrinovie D. S. (in cooperation with Vasie P. M.), “Analytic Inequalities”, Springer-
Verlag, Band 165, Berlin, 1970.
17. Mitrinovie D. S., Pecaric J. E., Fink A. M., “Classical and New Inequalities in
Analysis”, Kluwer Acad. Press., 1993.

 2. AN APPLICATION OF STIRLING’S FORMULA


Prof. SHAFIQUR RAHMAN, Bangladesh
Prof. LEONARD GIUGIUC, Dr. Tr. Severin

Professor Shafiqur Rahman discovered the following result:


ln(2 )
Theorem 0: n
lim n
n!  ne  ln n  .
2e 2e
That result appears to be easy to prove, but in fact it isn’t. Firstly, we will need two
of the most important results ever.
n n ( )
Theorem 1 (Stirling’s formula): n!  2n  ne  12n  n  1 , where n  (0, 1)
is called the Stirling constant.

Editura Cardinal 5
Theorem 2 (Lagrange’s mean value theorem): Consider the real numbers a, b such
that a  b and the function f : a, b  R . If f is continuous on a, b and differentiable on
(a, b ) then  c  (a, b ) for wich f b  f a  f  (c ).
( ) ( )
ba
We will not prove these classic results, they being well known by the reader.
The theorems 1 and 2 will be used for proving the next statemens:
Theorem 3: Let be the real numbers k, x  0 and the natural number m  1.
Then m x  k  m x  m m kx m1 .

n!  ne  (2n ) 2n  0.
1
Theorem 4: limn
n

Proof of theorem 3: We apply theorem 2 for f (t )  m t and the interval x, x  k and
m
xk  m x
get  m1 , where x  c  x  k. In conclusion, we have that:
k m c m1
m
xk  m x  m k .
m x m1
Proof of theorem 4: Via theorem 1 we have:
0  n n!  ne  (2n ) 2n  ne  n 2n  1  ne
n
 ne  n 2n , ( ) n  2.
1

12n
By theorem 4 for x  2n , k  1  ne and m  n, we get that
n
12n
2n  1  ne  n 2n  1  ne 
n n 1
n . In conclusion:
12n 12n n ( n1
n 2n )

2n  1  ne 2n  1  ne
n n n1
e 3  ne  n  1 , ( )n  2.
12n 12n n1
n ( 2n )
n

But lim 1  e n1


 1  0, so via squeeze theorem:
n 12n n n1
n n ( 2n )

n!  ne  (2n ) 2n  0.
1
lim
n
n

Now we are ready to prove the main theorem.


Proof of Theorem 0: We have:
n
n!  ne  ln n  n  (2n ) 2n1  n  ln n  n n!  n  (2n ) 2n1 ( ) n  2.
e e e
2e 2e
But ne  (2n ) 2n 
n  ln n  1  2n  (2n ) 2n1  2n  ln 2n  ln 2 ( ) n  2.
1

e 2e 2e
  1  x ln x
x

We’ll compute: lim


x .
x0 x
It is well known that lim x ln x  0 lim x x  1. (). So by l’ Hospital’s rule
x0 x0
 x  1  x ln x
 0  lim (ln   ln x  1 )  x  1  ln x .
x x
lim x
x0 0 x0

6 Revista de matematic[ din Craiova


(ln   1 )  x 
x x
Using ( ) lim
x0
 1  ln ; it remains lim
x0
ln x x 1 .

lim   e  1  0 . We apply again


e te
t t

Denote ln x  t. So the last limit becomes t


1 0
t
lim t 2  et ((t  1)ete  e  ete  e  ln ). But if P(t ) is a
t t t t
l’ Hospital’s rule and get t
lim P(t )  et  0. In conclusion t
lim t 2  e t (t  1 )e te   e  e te   e 
t t t t
polynomial then t
 x
 1  x ln x
x  ln . For x  1 , n  2,
 ln   0. We have been proved that: lim x
x0 2n
obtain that: lim
1
2n  (2n ) 2n  2n  ln 2n  ln . So lim n  (2n ) 2n  n  ln n 
1

n n e e 2e
ln(2 ) n 1
 . But lim n
n!  e  2n
( ) 2n
 0.
2e n
ln(2 )
So we can conclude that lim n
n!  ne  ln n  .
n 2e 2e
Remark: A beautiful consequence of this result is Lalescu’s theorem:
lim
n
n1
(n  1 )!  n n!  1e .

Indeed, if x n  n n!  ne  ln n , then lim (x n1  x n )  0. But lim (ln(n  1 )  ln n )  0


2e n n

so lim
n
n1
(n  1 )!  n n!  1e .

 3. EXISTEN|A }I CONSTRUC|IA
UNOR TRIUNGHIURI CLASICE
Prof. MARIAN DINC{, Bucure]ti

#n acest articol voi da o nou[ demonstra\ie teoremei reciproce a lui Dimitrie Pompeiu, care
se anun\[ astfel:
“Fiind dat un triunghi ABC, atunci exist[ un triunghi echilateral A1 B1 C1 ]i un punct M
situat @n planul s[u astfel @nc`t lungimile segmentelor MA1 , MB1 , MC1 s[ fie egale cu
lungimile laturilor triunghiului ABC”.
O demonstra\ie cu ajutorul numerelor complexe a fost dat[ de Aurel Angelescu @n lucrarea 1, iar o
demonstra\ie geometric[ a fost dat[ de autorul acestor r`nduri @n revista Arhimede unde fac o frumoas[
conexiune @ntre teorema reciproc[ a lui Dimitrie Pompeiu ]i teorema lui Torriceli.
Not[m BC  a, CA  b, AB  c ]i consider[m trei semidrepte ce au aceea]i
origine, punctul M. Fie MA 1  a, MB 1  b ]i MC 1  c ]i B 1 MC 1  A   ,
3
C 1 MA 1  B   , A 1 MB 1  C   , B 1 MC 1  C 1 MA 1  A 1 MB 1 
3 3

Editura Cardinal 7
 A    B    C    2 ]i vom ar[ta c[ triunghiul A 1 B 1 C 1 este echila-
3 3 3
teral. Folosim teorema cosinusului @n triunghiurile B 1 MC 1 , C 1 MA 1 , A 1 MB 1 ]i avem:

B1C1 2
 b 2  c 2  2bc  cos A    b 2  c 2  2bc  cos A  cos   sin A  sin  
3 3 3
a  b  c  4 3 aria(ABC )
2 2 2
 b 2  c 2  (a 2  b 2  c 2 )  1  3 bc  sin A  , (1 ), unde
2 2
am @nlocuit 2bc  cos A  b 2  c 2  a 2 ]i bc sin A  2 aria(ABC ).
Expresia de la (1) este simetric[ @n a, b, c ]i deci se va ob\ine aceea]i valoare pentru
C 1 A 1 2 ]i A 1 B 1 2 prin permut[ri circulare. A]adar: B 1 C 1  C 1 A 1  A 1 B 1 .
#n continuare voi solu\iona urm[toarea problem[ clasic[:
Problem[. Cu medianele unui triunghi ABC se poate construi un triunghi.
Solu\ie. Pentru demonstra\ie folosim aceea]i figur[, modific`nd unghiurile astfel
B 1 MC 1    A, C 1 MA 1    B, A 1 MB 1    C, iar lungimile segmen-
telor le @njum[t[\im: MA 1  a , MB 1  b ]i MC 1  c .
2 2 2
Voi demonstra c[ triunghiul A 1 B 1 C 1 are lungimile laturilor egale cu lungimile
medianelor triunghiului ABC. Folosim teorema cosinusului @n triunghiurile B 1 MC 1 ,
2 2
C 1 MA 1 , A 1 MB 1 ]i ob\inem: B 1 C1 2
 b  c 2 b c cos(  A ) 
2 2 2 2
2(b 2  c 2 )  a 2
 b  c  bc  cos A  b  c  2bc  cos A  b  c  b  c  a 
2 2 2 2 2 2 2 2 2

4 2 4 4 4
 (ma ) 2 ; deci B1 C1  ma .
L[s[m pe seama cititorului s[ arate c[ C 1 A 1  m b , repectiv A 1 B 1  m c.
Observa\ie. Punctul M este centrul de greutate al triunghiului A 1 B 1 C 1 .
Pentru a demonstra afirma\ia din observa\ia de mai sus va fi suficient s[ demon-
str[m c[: aria(B 1 MC 1 )  aria(C 1 MA 1 )  aria(A 1 MB 1 ).

Avem aria(B 1 MC 1 )  1  b  c  sin(  A )  bc  sin A 


aria(ABC )
]i @n mod
2 2 2 8 4
(
aria ABC )
analog: aria(C 1 MA 1 )  aria(A 1 MB 1 )  . Dac[ A 1 M  B 1 C 1  A 2 , atunci:
4
MA 2 aria(B 1 MC 1 )
  1 , deci M se afl[ la o treime de baz[ ]i dou[ treimi de v`rf.
A 1 A 2 aria(A 1 B 1 C 1 ) 3

Bibliografie.
1. Nicolae Mih[ileanu, Utilizarea numerelor complexe @n geometrie, Ed. Tehnic[, 1968.
2. Marian Dinc[, Marcel Chiri\[, Numere complexe @n matematica de liceu, Ed. All, 1998.

8 Revista de matematic[ din Craiova


 4. ASUPRA UNEI INEGALIT{|I

Prof. MARIAN CUCOANE}, M[r[]e]ti, Vrancea

Pentru orice a, b, c  0 are loc inegalitatea:


ab(a 2  b 2 ) bc(b 2  c 2 ) ca(c 2  a 2 )
   a2  b2  c2.
(a 2  c 2 )(b 2  c 2 ) (b 2  a 2 )(c 2  a 2 ) (c 2  b 2 )(c 2  a 2 )
#n continuare vom demonstra aceast[ inegalitate.
Vom folosi urm[torul rezultat cunoscut:
Lema 1 Fie ABC un triunghi oarecare ]i M un punct oarecare @n planul s[u. Dac[
not[m cu: BC  a; AC  b; AB  c; MA  R 1 ; MB  R 2 ; MC  R 3 atunci are loc inega-
R R R R R3R1
litatea: 1 2  2 3  ca  1.
ab bc
Pentru o demonstra\ie a acestei leme se poate consulta articolul 1. #n continuare
folosind lema 1 vom demonstra urm[torul rezultat:
Lema 2 Fie ABC un triunghi oarecare. Atunci are loc inegalitatea:
(p  b )(p  c ) (p  a )(p  c ) (p  a )(p  b )
a b ac c  p,
bc ab
nota\iile fiind cele obi]nuite.
Demonstra\ie #n lema 1 lu[m M  I centrul cercului @nscris @n triunghiul ABC.
Avem: R 1  AI  r A ; R 2  BI  r B ]i R 3  CI  r C astfel ob\inem:
sin 2 sin 2 sin 2
r2  r 2
 r 2
 1  a  sin A  b  sin B 
ab  sin A2  sin B2 bc  sin B2  sin C2 ac  sin A2  sin C2 2 2

c  sin C  a rb2  c  sin A  sin B  sin C ]i folosind egalit[\ile cunoscute:


2 2 2 2
a  b  c  sin A  sin B  sin C  p ]i sin A  (p  b )(p  c ) ]i analoagele, ob\inem
r2 2 2 2 2 bc
inegalitatea din lema 2. Revenim acum la demonstrarea inegalit[\ii ini\iale.
Fie x, y, z  0. Vom aplica inegalitatea din lema 2 pentru triunghiul ABC cu laturile
BC  a  y 2  z 2 , AC  b  x 2  z 2 , AB  c  x 2  y 2. Astfel ob\inem:
xy  (x 2  z 2 ) yz  (y 2  z 2 ) xz  (x 2  z 2 )
   x 2  y 2  z 2.
(x 2  z 2 )(y 2  z 2 ) (x 2  y 2 )(x 2  z 2 ) (x 2  y 2 )(y 2  z 2 )
Bibliografie
1. Marian Cucoane], “Generalizarea inegalit[\ii Stevin-Bottema”, REVISTA DE MATE-
MATIC{ DIN TIMI}OARA, nr. 4/Anul 2008, Pag. 3-5.
2. Art of Problem Solving.
*
* *

Editura Cardinal 9
****************************************************************** ***
* CONCURSURI }I OLIMPIADE *
*********************************************************************

OLIMPIADA NA|IONAL{ DE MATEMATIC{


Faza local[, Jud. Dolj
15.02.2015

Clasa a V-a
1. Numerele naturale m ]i n au proprietatea c[ (2 m  7 n ) se divide cu 5.
Ar[ta\i c[ num[rul (2 n  7 m ) se divide cu 5.
CRISTIAN MOAN|{
2. Determina\i mul\imea: M  abc  a  bc ]i b  ac sunt numere consecutive.
IONEL TUDOR, G.M. nr. 3/2013

3. Se consider[ num[rul:
A  1  2015  2015  2016  2015  2016 2  . . .  2015  2016 2015.
a) Ar[ta\i c[ num[rul a  1  2014  2014  2015 este p[trat perfect.
b) Ar[ta\i c[ num[rul b  1  2014  2014  2015  2014  2015 2 este cub perfect.
c) Ar[ta\i c[ A este p[trat perfect ]i cub perfect.
***
4. Determina\i numerele naturale a ]i b care verific[ rela\ia: a 4  5a  1  5 b .
GABRIEL TICA

Clasa a VI-a
1. Fie numerele naturale nenule a, b, c, x, y, z astfel @nc`t bcx  acy  abz  0 ]i
(a, b )  1, (b, c )  1, (c, a )  1. Ar[ta\i c[ a 2 b 2 c 2 divide (x 2  a 2 )  (y 2  b 2 )  (z 2  c 2 ).
VASILE SCURTU, G.M. nr. 11/2014
2. Numerele x  y, y  z, z  x sunt direct propor\ionale cu numerele 4, 6, 8.
xy  xz  yz
a) Afla\i valoarea raportului: 2 .
x  y2  z2
b) Dac[ a, b, c  1, 2, . . ., 9, a  b  c  a, s[ se determine valorile maxime ]i
axy  bxz  cyz
minime ale raportului: .
x2  y2  z2
***

10 Revista de matematic[ din Craiova


3. Fie unghiul XOY ]i numerele naturale distincte a, b. Pe (OX consider[m @n
ordine punctele A 1 , A 2 , A 3 , . . . astfel @nc`t OA 1  A 1 A 2  A 2 A 3  . . .  a, iar pe OY
consider[m @n ordine punctele B 1 , B 2 , B 3 , . . . astfel @nc`t OB 1  B 1 B 2  B 2 B 3  . . .  b.
a) Ar[ta\i c[ exist[ A  A 1 , A 2 , A 3 , A 4 , . . . ]i B  B 1 , B 2 , B 3 , B 4 , . . . astfel
@nc`t triunghiul OAB este isoscel.
b) Ar[ta\i c[ exist[ A, C  A 1 , A 2 , A 3 , A 4 , . . . ]i B, D  B 1 , B 2 , B 3 , B 4 , . . .
astfel @nc`t: OAD  OBC.
STELU|A ]i MIHAI MONEA

4. Pe dreapta d, se consider[ punctele O, A, B, C, D, E, F @n aceast[ ordine, astfel


@nc`t AB  2  OA, B este mijlocul lui AC, C este mijlocul lui BD, D este mijlocul
lui BE ]i E este mijlocul lui BF. S[ se arate c[:
a) segmentele (AE, CD ), respectiv (AD, BC ) au acela]i mijloc;

b) AC  AB  BC  OA  CF .
BE AD EF DE OE
***

Clasa a VII-a
1. S[ se rezolve @n N ecua\ia:
1 1 1 2
  ...   ( 101  9 ).
2
1  12  1 2  22  1 n  n2  1
***
2. Fie x 1 , x 2 , . . ., x2015  Z, astfel ca x 1  x 2  . . .  x 2015  2. Se noteaz[
S  x 1  x 2  . . .  x 2015 , P  x 1  x12  x 2  x13  . . .  x 2015  x11 .
a) S[ se arate c[ S este un num[r @ntreg divizibil cu 2 sau cu 4.
b) Calcula\i produsul P.
CRISTIAN MOAN|{

3. Se consider[ paralelogramul ABCD ]i punctele M ]i N situate pe latura (CD ),


respectiv (AB ). Demonstra\i c[: A AMB  A CND  A ABCD .
***
4. Fie p[tratul ABCD. Pe latura AB se construie]te @n exterior triunghiul echilate-
ral ABE. Perpendiculara din A pe dreapta DE intersecteaz[ perpendiculara din B pe
dreapta CE @n punctul F. Ar[ta\i c[ patrulaterul BEFC este romb.
***

Editura Cardinal 11
Clasa a VIII-a
1. a) Fie a, b, c  R. Demonstra\i c[:
(a  b  c) 2  (a  b  c ) 2  (a  b  c ) 2  (a  b  c) 2  4(a 2  b 2  c2 ).
b) S[ se demonstreze c[ num[rul A  22 22  44 44  66 66 se poate scrie ca o sum[
de patru numere naturale nenule p[trate perfecte.
***
2. Determina\i mul\imea I  , a  a , , unde a  R .
1 

***
3. Fie M un punct exterior planului trapezului ABCD (AD  BC ), iar E, F proiec-
\iile lui pe dreptele AD, respectiv BC.
a) Demonstra\i c[ (MEF )  (ABC );
b) Dac[ distan\ele de la M la bazele trapezului sunt de 6 3 cm, iar m[sura
diedrului format de planele (MAD ) ]i (MBC ) este de 60  , calcula\i distan\a de la M la
planul (ABD ).
***
4. Se consider[ cubul ABCDMNPQcu lungimea muchiei egal[ cu 5. Planul (ANQ )
intersecteaz[ planele (MBC ), (MCD ), (MDB ) dup[ dreptele d 1 , d 2 , respectiv d 3 .
a) Ar[ta\i c[ dreptele d 1 , d 2 , d 3 sunt concurente dou[ c`te dou[.
b) Demonstra\i c[ aria triunghiului format de cele trei drepte este mai mic[
dec`t 2.
***

Clasa a IX-a
1. Pentru x, y  R cu proprietatea c[ 2x  y  10 se consider[ expresia:
x  y  x  2y  . . .  x  404y
E(x, y )  ,
404
unde a reprezint[ partea @ntreag[ a num[rului real a. Ar[ta\i c[:
404(x  y )  E(x, y )  2015.
DAN SECL{MAN

2. S[ se determine n  N astfel @nc`t: n 2  89n  2011  N.


PETRIC{ DICU, G.M. nr. 6/2011

3. S[ se determine o progresie aritmetic[ @n care suma primilor n termeni este


egal[ cu 3n2  4n, () n  N. Unii termeni ai progresiei sunt p[trate perfecte. S[ se
determine o expresie general[ a acestor termeni ]i s[ se calculeze primii 6 termeni.
***

12 Revista de matematic[ din Craiova


4. Fie patrulaterul ABCD, H 1 ortocentrul triunghiului ABC ]i H 2 ortocentrul tri-
unghiului DBC. Ar[ta\i c[ ABCD este inscriptibil dac[ ]i numai dac[ H 1 H 2  AD.
***

Clasa a X-a
1. S[ se rezolve @n R ecua\iile:
a) 2x1 1 x1  1  3.
2  3  2  6 2 2x  2 x2  3 2 x
b) 3 2x  1  3 4x  5  3 5x  3  3 11x  3 .
***
2. Fie a, b, c  1, astfel @nc`t abc  8. Demonstra\i c[:
log ab (a  b )  log bc (b  c )  log ca (c  a )  3.
C{T{LIN CRISTEA
3. Fie A 1 A 2 . . .A n un poligon regulat, @nscris @n cercul C(O, R ) ]i M  C(O, R ).
Ar[ta\i c[: nR  MA 1  MA 2  . . .  MA n  2nR.
***
4. a) Fie f : R  R o func\ie cu proprietatea:
f (f (x ))  2014  f (x )  x 2015 , () x  R.
Demonstra\i c[ f este injectiv[.
b) Exist[ func\ii f : (1, ) R, injective, astfel @nc`t
f (x )  f (2 x )  f log 2 x  2015, () x  (1, )?
***

Clasa a XI-a
a 0 b
1. Fie matricea A  0 c 0 , unde a, b, c  R.
b 0 a
a) S[ se calculeze An , n  N .
b) Dac[ a, b, c sunt lungimile laturilor unui triunghi oarecare cu a  b, s[ se
An
calculeze lim
n B n
, unde A n ]i B n reprezint[ suma elementelor de pe diagonala princi-
pal[, respectiv de pe diagonala secundar[ a matricei A n .
***
2. Fie ]irul (a n ) n1 , definit prin: a n  2 1  1  ...  2 1 .
n  1 n2  2 n n
a) Ar[ta\i c[: a n  1, ( ) n  1.
b) Calcula\i: lim
n
(n  a n ).
n
c) Calcula\i: lim
n
( n  an ) .
***

Editura Cardinal 13
3. Fie A  M 2 (R ), astfel @nc`t Tr(A )  1 ]i det(A 2  A  I 2 )  2 3  det A.
Demonstra\i c[: det(A 2  3  I 2 )  3 .
DAN SECL{MAN
4. Fie ]irul (x n ) n1 strict descresc[tor @n care
x 1  0 ]i x 4n  4x n  x 2n  x 3n  x n1  4, ( ) n  1.
Ar[ta\i c[ ]irul (x n ) n1 este convergent ]i afla\i limita sa.
GABRIEL TICA, G.M. nr. 9/2008

Clasa a XII-a
1. Fie func\ia f : Z 9  Z 9 . S[ se determine toate submul\imile nevide A ale lui Z 9
cu proprietatea f (A )  A.
BENEDICT G. NICULESCU, G.M. nr. 9/2014
2 1
2. Fie A  ]i G  X a  M 2 (R )  X a  I 2  aA, a  1.
2 1
Demonstra\i c[ G este grup abelian @n raport cu @nmul\irea matricelor, izomorf cu
grupul aditiv al numerelor reale.
***
3. Dac[ f : R  R este o func\ie care admite primitive, demonstra\i c[ pentru orice
primitiv[ F a sa exist[ c  R cu proprietatea:
f (c )  sin F(c )  c 2015 .
DAN SECL{MAN
4. Calcula\i:
t 2 1
t 3  1 dt 1
a)  b) lim 
t2 dt
; .
3
t2  1 x0 x
t 3
t2  1  t(t 2  1 ) 
3 2 t 2 1
3 t

***

*
* *

14 Revista de matematic[ din Craiova


************************************************************ *********
* PROBLEME PROPUSE *
*********************************************************************

CICLUL PRIMAR
Clasa I
I.1. }tefania ]i D[nu\a au plantat @n gr[dina ]colii 30 de flori: 13 lalele, narcise ]i 8
zambile. C`te narcise au plantat feti\ele?
I.2. Teodora ador[ c[r\ile ]i revistele pentru copii. Ea are o bibliotec[ de care e tare
m`ndr[. A str`ns @n micu\a ei bibliotec[: 30 de c[r\i cu pove]ti, 20 de c[r\i cu poezii ]i
45 reviste interesante pentru copii.
C`te c[r\i ]i reviste are Teodora @n total @n bibliotec[?
I.3. Irinel, Vasilic[, Georgian, Alexandru ]i Cristian au organizat o curs[ de ma]i-
nu\e. Irinel a concurat cu 9 ma]inu\e, Vasilic[ cu 11, Georgian ]i Alexandru cu c`te 5,
iar Cristian cu o ma]inu\[. C`te ma]inu\e au participat la curse?
I.4. Pe o r[muric[ c`nt[ o mic[ r`ndunic[. A c`ntat frumos ]i-au venit s[ o asculte
un motan ]i o pisic[, 3 r[\oi, vreo dou[ g`]te, Azorel ]i 2 cocori. C`te vie\uitoare se
bucur[ de-a r`ndunicii c`ntare?
I.5. Cu c`t este mai mic num[rul 17 dec`t r[sturnatul num[rului 74, c[ruia I s-a
ad[ugat cel mai mare num[r par de o cifr[?
I.6. Eu, sora ]i p[rin\ii mei avem @mpreun[ 90 ani. C`t @nsumau v`rstele noastre
acum 3 ani?
I.7. Veveri\a avea luni 83 alune. Mar\i a m`ncat 24. Miercuri a cules 19. Joi a
primit 12. Vineri a m`ncat c`teva. Acum are 79 alune. C`te alune a m`ncat vineri?
I.8. M[ g`ndesc la un num[r, @i adaug 16, scad apoi 3 ]i ob\in 25. La ce num[r
m-am g`ndit?
I.9. Bunicul a cump[rat 2 kilograme de mere. El se g`nde]te: 7 mere o s[ folo-
seasc[ bunica pentru tort, 3 le vor m`nca nepo\ii, le mai p[strez 3 pentru pachetul de
m`ine de la ]coal[ ]i mai r[m`n 4. C`te mere au intrat @n cele 2 kilograme?
I.10. Elevii de la clasele I vor organiza o expozi\ie de m[r\i]oare. Clasa I A confec-
\ionat 30 de m[r\i]oare, clasele I B, I C ]i I D, c`te 20, iar clasa I E 15 m[r\i]oare.
C`te m[r\i]oare vor fi expuse?
I.11. Miruna a primit de la mama ei 6 bomboane, iar de la bunica @nc[ 12. A m`n-
cat 4, iar colegei de banc[ i-a oferit 2. C`te mai poate m`nca?
I.12. Scrie\i @n ordine descresc[toare numerele naturale de dou[ cifre pentru care
suma dintre cifra zecilor ]i cifra unit[\ilor este 10.

Editura Cardinal 15
I.13. Zece pui]ori mergeau @n ]ir, unul c`te unul. Cel negru avea @naintea lui 5
pui]ori albi. C`\i pui]ori erau @n urma lui? Care poate fi num[rul maxim de pui]ori
albi din ]ir? Dar num[rul minim?
I.14. Alex a primit de la bunica: 2 banane, 2 pere, 3 portocale, 5 prune, 4 piersici ]i
2 mere. C`te fructe din livada bunicii a primit Alex?
I.15. Calculeaz[ suma vecinilor pari ai num[rului 4.
I.16. Suma a 3 numere este 8. S[ se afle fiecare num[r ]tiind c[ suma primelor
dou[ este 5, iar a ultimelor dou[ 6.
I.17. #ntr-un autobuz erau 4 c[l[tori. La prima sta\ie urc[ 3 c[l[tori ]i coboar[ 1, la
urm[toarea sta\ie urc[ 4 ]i coboar[ 2, la a treia urc[ 2 ]i coboar[ 3. C`\i c[l[tori
coboar[ la a patra sta\ie care este ]i ultima?
I.18. Alina are 16 ani, fiind cu 3 ani mai mare dec`t sora ei, Mihaela. Cu c`\i ani va
fi mai mic[ Mihaela dec`t Alina peste 3 ani?
I.19. Cu c`t este mai mare vecinul cel mic al celui mai mare num[r natural de dou[
cifre dec`t num[rul 8?
I.20. C`te ou[ a cump[rat bunica dac[ a folosit la o pr[jitur[ 5, din gre]eal[ a spart
unul ]i i-au mai r[mas 42?
I.21. Irina are 5 ani iar fratele ei cu 1 an mai mult. C`\i ani aveau @mpreun[ cei doi
fra\i acum 1 an?
I.22. Afl[ diferen\a dintre cel mai mare num[r natural scris cu dou[ cifre diferite ]i
cel mai mic num[r natural scris cu dou[ cifre identice.
I.23. Scrie toate numerele naturale cu suma cifrelor egal[ cu 9.
I.24. Dac[ dintr-o l[di\[ se iau 2 mere ]i se pun @n alt[ l[di\[, atunci @n fiecare l[-
di\[ r[m`n c`te 35 de mere. C`te mere au fost la @nceput @n fiecare l[di\[?
I.25. M[ g`ndesc la un num[r, @l adun cu 30 ]i ob\in 70. La ce num[r m-am g`ndit?

Clasa a II-a
II.1. Pe o ramur[ de m[r erau 19 mere, iar pe alta 17 mere. Dup[ ce Mihai ]i Ionu\
au cules c`te un num[r egal de mere, pe cele dou[ ramuri au mai r[mas 18 mere.
C`te mere a cules Mihai?
II.2. #n dou[ cutii erau 57 de cuburi. Dup[ ce Daniel ia din prima cutie 14 cuburi,
iar Ciprian ia din a doua cutie 13 cuburi, @n fiecare cutie [m`n tot at`tea cuburi.
C`te cuburi au fost la @nceput @n fiecare cutie?
II.3. #n clasele @nt`i ]i a doua sunt 53 de elevi dintre care 27 sunt b[ie\i. }tiind c[ @n
clasa a doua sunt 15 fete, afla\i @n care clas[ sunt mai multe fete ]i cu c`te.
II.4. Afla\i dublul sumei numerelor a, b, c ]i d, dac[:
a 104  d, c 58  b, 251  180  d, b 169  a.

16 Revista de matematic[ din Craiova


II.5. Elimina\i o cifr[ din num[rul 719, f[r[ a schimba ordinea cifrelor, astfel @nc`t
s[ r[m`n[ un num[r de dou[ cifre: a) c`t mai mare; b) c`t mai mic.
II.6. C`nd s-a n[scut Raluca, mama ei avea 25 de ani. Raluca are acum 9 ani.
C`\i ani va avea mama ei peste 8 ani?
II.7. Din suma numerelor de trei cifre distincte scrise cu numerele 3, 0 ]i 1 sc[de\i
r[sturnatul sumei ob\inute.
II.8. Din ce num[r trebuie sc[zut 135 pentru a ob\ine un num[r cu 10 mai mic
dec`t 150?
II.9. Calcula\i suma primelor patru numere pare consecutive scrise cu patru cifre.
II.10. Pe fiecare dintre cele trei rafturi ale bibliotecii clasei erau 15, 20 respectiv 25
c[r\i. Au mai fost procurate @nc[ 30 c[r\i. Cum pot fi repartizate cele 30 de c[r\i, astfel
@nc`t pe fiecare raft s[ fie acela]i num[r de c[r\i?
II.11. Mihai a m`ncat 8 bomboane ]i a constatat c[ acest num[r reprezint[ un sfert
din totalul bomboanelor din cutie. C`te bomboane are cutia?
II.12. Dup[ ce Ana i-a dat surorii ei 26 lei, cele dou[ fete aveau aceea]i suma de
bani. Cu c`t a avut mai mul\i bani Ana dec`t sora ei?
II.13. Se @nt`lnesc 8 b[ie\i. C`te str`ngeri de m`n[ au loc, dac[ fiecare copil d[
m`na o singur[ dat[ cu ceilal\i?
II.14. Afl[ suma a dou[ numere ]tiind c[ dac[ se m[re]te primul cu 13 ]i al doilea
cu 9, suma devine 91, iar dac[ se mic]oreaz[ cu acelea]i numere se ob\ine primul
num[r. Afl[ numerele.
II.15. Alin, Daria ]i Cristian au @mpreun[ tot at`tea c[r\i c`t r[sturnatul num[rului
18. }tiind c[ Alin ]i Daria au @mpreun[ 56 c[r\i, iar Cristian are cu 8 mai multe dec`t
Daria, afla\i c`te c[r\i are fiecare copil.
II.16. M[ g`ndesc la un num[r. Scad din el de dou[ ori num[rul piticilor din
povestea “Alba ca z[pada” ]i ]apte pitici ]i ob\in de trei ori num[rului iezilor din
povestea “Capra cu trei iezi”. La ce num[r m-am g`ndit oare?
II.17. La o libr[rie s-au adus 342 de mingi mari, mijlocii ]i mici. Dintre acestea,
167 nu erau mingii mici ]i 211 nu erau mingii mari. C`te mingi erau de fiecare fel?
II.18. La ora de sport copiii s-au a]ezat pe 3 r`nduri astfel: pe al doilea r`nd erau
13 copii, adic[ cu 9 mai mul\i dec`t pe primul r`nd ]i cu 4 mai pu\ini dec`t cei de pe al
treilea r`nd. C`\i copii erau @n total?
II.19. Din suma numerelor pare cuprinse @ntre 24 ]i 36 sc[de\i r[sturnatul celui mai
mare num[r de dou[ cifre distincte.
II.20. Mirela are cu 13 lei mai pu\in ca Vali. Dac[ ea ar cump[ra un caiet cu 8 lei,
atunci ar avea jum[tate din suma lui Vali. C`\i lei au ei @n total?
II.21. Cristi are cu 28 lei mai mult ca sora sa. Dac[ sora sa i-ar da lui Cristi 10 lei,
acesta ar avea dublu fa\[ de ea. C`\i lei are fiecare?
II.22. Dac[ la un num[r ad[ug[m 13, apoi dubl[m aceast[ sum[ ob\inem 46.
Afl[ num[rul ini\ial.

Editura Cardinal 17
II.23. Dac[ m[rim un num[r cu 20 observ[m c[ este cu 13 mai mare dec`t altul ]i
dublu fa\[ de el. Care sunt numerele?
II.24. #n anul 2006, sora mea avea v`rsta egal[ cu suma cifrelor num[rului 2006.
#n c`\i ani s-a mai @nt`mplat acest lucru din anul 2000 p`n[ @n prezent?
II.25. Marian are 2 fra\i ]i dou[ surori. C`\i copii sunt @n familie?

Clasa a III-a
III.1. C`nd Dan avea v`rsta de 11 ani, sora sa avea 8 ani. Acum au @mpreun[ 51 de
ani. Calculeaz[ v`rsta pe care o are fiecare acum.
III.2. #n vacan\a de iarn[ Andrei a rezolvat 48 de probleme. Dac[ ar mai fi rezolvat
12 probleme, ar avea rezolvate de 4 ori mai multe dec`t fratele lui.
C`te probleme a rezolvat fratele lui?
III.3. Suma a trei numere este 42. Dac[ dubl[m primul num[r ob\inem pe al doilea,
iar dac[ din al treilea num[r @l sc[dem pe primul ob\inem pe al doilea.
Care sunt cele trei numere?
III.4. Suma a trei numere naturale pare consecutive este egal[ cu 30. Afla\i cele
trei numere naturale.
III.5. #n dou[ cutii sunt c`te 33 de bomboane. Dac[ iau din prima cutie 8 bombo-
ane ]i le pun @n a doua, c`te bomboane vor fi @n fiecare cutie?
III.6. #ntr-un co] erau 70 kg de mere, iar @n altul cu 29 kg mai pu\ine. Din primul
co] se consum[ 32 kg, iar din al doilea cu 4 kg mai multe. C`te kilograme de mere au
r[mas @n cele dou[ co]uri?
III.7. Alin ]i Andrei rezolv[ probleme pentru un concurs de matematic[. Dac[ Alin
ar mai rezolva @nc[ 30, ar rezolva c`t Andrei. Dac[ Andrei ar mai rezolva @nc[ 40,
atunci ar rezolva dublu fa\[ de Alin. C`te probleme a rezolvat fiecare copil?
III.8. Un elev rezolv[ @n prima zi 3 probleme, @n a doua 6, ]i tot a]a, @n fiecare zi
dintr-o s[pt[m`n[, C`t dublul celor lucrate @n ziua precedent[. C`te probleme a rezol-
vat @n total?
III.9. #n anul 2009 suma v`rstele membrilor unei familii compus[ din: mama, tata
]i doi copii este de 84 de ani. Diferen\a v`rstelor p[rin\ilor este de 6 ani. Copiii lor au
9, respectiv, 11 ani. Ce v`rst[ au p[rin\ii? #n ce an s-a n[scut fiecare?
III.10. O gr[din[ are forma unui triunghi cu laturile de m[rimi diferite. C`\i m de
gard se folosesc pentru @mprejmuirea gr[dinii, ]tiind c[ m[rimile laturilor sunt trei
numere consecutive, iar primul este 205?
III.11. Cu c`t este mai mare suma dec`t diferen\a numerelor impare cuprinse @ntre
488 ]i 492?

18 Revista de matematic[ din Craiova


III.12. M[ g`ndesc la un num[r. #l @mpart la 7, iar rezultatul ob\inut @l @nmul\esc cu
8. #i adaug apoi produsul ob\inut @ntre c`turile numerelor 72 ]i 9 ]i 54 ]i 6, iar rezulta-
tul final este 138. La ce num[r m-am g`ndit?
III.13. Veveri\a poate tr[i 7 ani, iepurele cu 3 ani mai mult dec`t ea, vulpea @nc[ o
dat[ c`t iepurele, cerbul cu 10 ani mai mult dec`t vulpea, iar ursul c`t vulpea ]i cerbul
la un loc. C`\i ani tr[ie]te ursul?
III.14. Care este cel mai mare num[r de ordinul 3 care are cifra zecilor egal[ cu
suma dintre cifra unit[\ilor ]i a sutelor?
III.15. Afl[ cel mai mare num[r de ordinul 5 ]tiind c[ produsul dintre cifra sutelor
]i cifra unit[\ilor este 16.
III.16. Maria are o sum[ de bani. Mama @i mai d[ 17 lei, iar tata @nc[ 35 lei. Maria
le d[ celor doi fra\i ai ei c`te 23 de lei ]i prime]te de la bunica un num[r ce reprezint[
diferen\a dintre sumele primite de la cei doi p[rin\i. Acum, fata are 43 de lei.
C`\i lei a avut la @nceput?
III.17. Din diferen\a numerelor 156 ]i 76, scade p[trimea lui 84 m[rit[ cu dublul
lui 16 ]i ob\ii astfel triplul lui m. Care este m?
III.18. Andrei are 54 de baloane ]i 36 de ma]inu\e. #n fiecare zi el sparge c`te 4
baloane ]i prime]te 2 ma]inu\e. #n c`te zile Andrei va avea tot at`tea baloane c`t
ma]inu\e?
III.19. #ntr-o p[dure sunt stejari, fagi ]i salc`mi. Afla\i c`\i copaci sunt din fiecare
fel, ]tiind c[ fagi ]i stejari sunt 797, stejari ]i salc`mi sunt 816, iar fagi ]i salc`mi sunt
671.
III.20. Jum[tatea unui num[r este cu 639 mai mic[ dec`t @ndoitul num[rului. Care
este acel num[r?
III.21. Un sfert din jum[tatea distan\ei dintre dou[ localit[\i este 121. Care este
distan\a dintre cele dou[ localit[\i?
III.22. Doi copii au @mpreun[ 34 lei. Al doilea are cu 7 lei mai mult dec`t jum[tate
din c`t are primul. C`\i lei are fiecare?
III.23. Daniela cheltuie]te jum[tate din suma pe care o are pentru a-]i cump[ra o
jachet[, iar o cincime din rest pentru a-]i cump[ra o bluz[. }tiind c[ din banii r[ma]i
]i-ar mai putea cump[ra 4 c[r\i a 5 lei ]i un stilou cu 60 lei, s[ se afle suma ini\ial[.
III.24. De@mp[r\itul este un num[r de forma abcd, @mp[r\itorul ]i c`tul sunt nu-
mere de forma xy, @n care x  y  9, iar restul este 89. Reconstitui\i @mp[r\irea.
III.25. Determina\i num[rul natural abc ]tiind c[ prima cifr[ este @ntreitul celei
de-a doua, dar jum[tatea celei de-a treia.

Editura Cardinal 19
Clasa a IV-a
IV.1. Determina\i numerele naturale de forma abc ]tiind c[ a  b  10.
IV.2. a) Afla\i c`te numere naturale de 5 cifre @ncep cu 53.
b) Afla\i c`te numere naturale de 5 cifre se termin[ cu 53.
c) Afla\i c`te numere naturale de 5 cifre distincte @ncep cu 53.
IV.3. Afla\i numerele naturale xyz astfel @nc`t: xyz  zyx  198.
IV.4. Reconstitui\i adunarea: abc  bc  c  1081.
IV.5. Suma a ]ase numere naturale diferite de zero este 19. Ar[ta\i c[ cel pu\in 2
numere sunt egale.
IV.6. Determina\i numerele naturale nenule care @mp[r\ite la 1986 dau c`tul de
cinci ori mai mic dec`t restul.
IV.7. Fie num[rul x  1234567891011121314 . . .19601961. Determina\i cel mai
mic num[r posibil dup[ @nl[turarea a 85 de cifre.
IV.8. Afla\i v`rstele lui Ion ]i Vasile, ]tiind c[:
a) @n urm[ cu un an, Vasile avea v`rsta de trei ori mai mare dec`t diferen\a
dintre v`rsta lui ]i a lui Ion;
b) peste doi ani, suma v`rstelor lor va fi de trei ori mai mare dec`t v`rsta
actual[ a lui Ion.
IV.9. a) Determina\i toate numere naturale n care au proprietatea c[ n  2010  4n.
b) Calcula\i suma ]i produsul tuturor numerelor naturale determinate la punc-
tul a).
IV.10. Maria ]i-a propus s[ citeasc[ o carte. Dac[ ar citi c`te 15 pagini pe zi, ar
termina-o @ntr-un anumit num[r de zile. Maria cite]te @ns[ doar @n prima zi 15 pagini,
iar @ncep`nd cu a doua zi cite]te c`te 20 pagini pe zi. Astfel, ea termin[ cu dou[ zile
mai devreme dec`t ]i-a propus. C`te pagini are cartea?
IV.11. Ioana culege o l[di\[ de c[p]uni @n 40 minute, iar Luiza culege o l[di\[ @n 2
ore. #n c`t timp vor culege @mpreun[ 3 l[di\e de c[p]uni?
IV.12. Mama are cu 14 lei mai mult dec`t Paul ]i cu 10 lei mai mult dec`t Tudor.
C`\i lei va da fiec[ruia din b[ie\i pentru a avea to\i trei aceea]i sum[?
IV.13. Robert are cu 60 lei mai mult dec`t fratele s[u. Dup[ ce cheltuie 40 lei,
constat[ c[ are de trei ori mai mult dec`t fratele. C`\i lei avea Robert la @nceput?
IV.14. Dan a]eaz[ creioanele @n cutii. Dac[ pune c`te 4 @n fiecare cutie, @i r[m`n 2
creioane. Dac[ pune c`te 6 @n fiecare cutie, @i r[m`ne o cutie goal[ ]i una cu 4
creioane. C`te cutii ]i c`te creioane are Dan?
IV.15. Suma a trei numere este 134. Dac[ se @mparte primul num[r la al doilea se
ob\ine c`tul 13 ]i restul 4, iar dac[ se @mparte al doilea num[r la al treilea, se ob\ine
c`tul 2 ]i restul 1. Afla\i numerele.
IV.16. Se dau 3 numere naturale a, b, c. }tiind c[: a  b  25 ]i b  2c  25, afla\i
suma 2a  5b  6c.

20 Revista de matematic[ din Craiova


IV.17. Se dau numerele:
a  1  2  3  . . .  9; b  11  22  33  . . .  99; c  111  222  333  . . .  999.
S[ se calculeze c`tul @mp[r\irii lui a  b  c la 123.
IV.18. #mp[r\ind num[rul natural a la num[rul natural b, ob\inem c`tul 6 ]i restul
31. S[ se afle a ]tiind c[ a  b  196.
IV.19. #ntr-un co] sunt 28 fructe: mere, pere ]i caise. C`te fructe sunt de fiecare fel,
dac[ mere sunt de 6 ori mai multe dec`t pere, iar @n co] se afl[ cel pu\in un fruct de
fiecare fel?
IV.20. Afla\i c`\i termeni are ]irul urm[tor:
a) 19, 20, 21, . . . .41, 42, 43, . . . . . . . . . ., 63.
b) 98, 99, 100, . . . . . . . . , 124.
c) 33, 44, 55, . . . . . . . . ., 110.
IV.21. Narcis a rezolvat @ntr-o s[pt[m`n[ 1/3 ]i @nc[ 3 probleme din tema de
vacan\[, a doua s[pt[m`n[ 1/3 ]i @nc[ 3 probleme ]i i-au mai r[mas 123 de probleme.
C`te probleme a avut de rezolvat pe vacan\[?
IV.22. Suma a 4 numere este 1707. Primul num[r este cu 2 mai mare dec`t al
doilea, al treilea este de 3 ori mai mic dec`t al doilea ]i al patrulea este cu 3 mai mare
dec`t primul num[r. Care sunt cele 4 numere?
IV.23. #n trei depozite sunt 1680 kg porumb. Dac[ @n primul ar fi de 4 ori mai
mult, iar @n al doilea de 2 ori mai mult, ar avea fiecare o cantitate egal[ cu c`t este @n al
treilea. Ce cantitate este depozitat[ @n fiecare depozit?
IV.24. C`tul @mp[r\irii a dou[ numere este 3, iar restul 10. Dac[ adun[m
de@mp[r\itul, @mp[r\itorul, c`tul ]i restul ob\inem 443. S[ se afle cele dou[ numere.
IV.25. Diferen\a a dou[ numere este 641. Dac[ le @mp[r\im ob\inem c`tul 7 ]i
restul 11. Afla\i suma celor dou[ numere.

Au colaborat la realizarea acestui num[r:

DORINA BR#NDU}E - }c. Gen. Nr. 3, Cugir, DANIELA DR{GOESCU - }c. Gen.
“G. Usc[tescu”, Tg. C[rbune]ti, CAMELIA BULIGIU - }c. Gen. “G. Usc[tescu”, Tg.
C[rbune]ti, MARIANA C#L|ARU - }c. Gen. “G. Usc[tescu”, Tg. C[rbune]ti,
LOREDANA ELENA PETRIC{ - }c. Gen. “G. Usc[tescu”, Tg. C[rbune]ti, DORIN
BR#NDU}E - }c. Gen. Nr. 3, Cugir, NICOLETA CHI|ORAN - Rovinari, Gorj, RODICA
CRAU - }c. Gen. “G. Usc[tescu”, Tg. C[rbune]ti, SILVIA AVRAMESCU - }c. Gen. “G.
Usc[tescu”, Tg. C[rbune]ti, GHERGHINA LUNGEANU - Gala\i, SIMONA-ELIZA
MAZILU - Rovinari, Gorj, ADRIANA C#RSTEA - Tg. C[rbune]ti, LILIANA
ARGENTOIANU - C. N. “Fra\ii Buze]ti”, Craiova, ELENA CIOLACU - }c. Gen.
"Grigore Geam[nu", Turcine]ti, Gorj, GABRIELA MATI} - }c. Gen. Nr. 26, Timi]oara,
DORINA BU}OI - }c. Gen. Bor[scu, Gorj, ANDA TEC}A - }c. cu cls. I-VIII Nr. 3, Cugir,
CORINA ELENA BIANCHI - }c. Gen. “Arcani”, Arcani, ELENA CRI}AN - }c. Gen. Nr.
3 Cugir, DANIELA ANDREESCU - }c. Gen. "Grigore Geam[nu", Turcine]ti, Gorj,
LILIANA VASILESCU - }c. Gen. “G. Usc[tescu”, Tg. C[rbune]ti, ELENA POPESCU -
Lic. cu Prog. Sportiv, Tg. Jiu, ELENA NICOLAE - }c. Gen. Ple]oi, Dolj,
IONELIA-SIMONA NAE - Booveni, Dolj, MARIANA Z{LOG - Tg. Jiu, SIMONA
CROITORU - Gureni, Dolj, IRINA BARBU - Craiova, IOANA PI|A - Vi]ina, Olt,

Editura Cardinal 21
AUREL }TEFAN - C. N. “Elena Cuza”, Craiova, MARIAN CIUPERCEANU - C. N.
“Fra\ii Buze]ti”, Craiova, LETI|IA DR{GHICI - C. N. “Fra\ii Buze]ti”, Craiova,
VERONICA PRUFU - C. N. “Fra\ii Buze]ti”, Craiova, CRISANDA-GEORGIA TURCU -
C. N. “Fra\ii Buze]ti”, Craiova, IONELA BARB{RAS{ - C. N. “Fra\ii Buze]ti”, Craiova,
BADEA ADELA - Craiova, VILIANA CONSTANTIN - Craiova, LAURA ZAHARIA -
Craiova, MIRELA SIMONA POPESCU - C. N. “Elena Cuza”, Craiova, ROXANA
POPESCU - Craiova, LAVINIA VULPE - Craiova, DIANA BORANGIC - Craiova,
LILIANA GENTOIANU - Craiova, EUGENIA PETCU - }c. Gim. "Gh. |i\eica", Craiova,
PETCU VIORICA ANGELICA - }c. Primar[ "Vasile Carabis", C@mpofeni, Gorj, IONICA
TRIC{ - }c. Primar[ P[i]ani, Stoina, Gorj, RODICA CRAU - }c. Gen. "George Usc[tescu",
Tg. C[rbune]ti, ALINA VL{DU|ESCU - }c. Gim. "George Usc[tescu", Tg. C[rbune]ti,
VALENTINA-IRINA COVERC{ - }c. Gim. Ureche]ti, Ureche]ti, Gorj, ANA PAN{ - }c.
Gen., Cru]e\, Gorj, GABRIELA MATI} - }c. Gim. Nr. 26, Timi]oara, ECATERINA
BOGDAN - C. N. "Carol I", Craiova, LICA IONEL OVIDIU - C. N. "Carol I", Craiova,
MARIA DORAN - C. N. "Carol I", Craiova, ION ROTARU - C. N. "Carol I", Craiova,
COCA TANCIU - C. N. "Carol I", Craiova, CATARGENA RADA - C. N. "Carol I",
Craiova, MIHAELA LIC{ - C. N. "Elena Cuza", Craiova, DINU CONSTANTINA - }c. cu
clasele I-VIII, Castranova, CORINA MIRELA ENEA -C. N. "Carol I", Craiova, LIANA
VELISCU - C. N. "Carol I", Craiova, ISABELLA NICA - C. N. "Carol I", Craiova, IOANA
PI|A - Vi]ina, Olt, ELENA GRIGORE - C. N. "Fra\ii Buze]ti", Craiova, LAURA
ZAHARIA - C. N. “Elena Cuza”, Craiova, ELIZA CHILOM - "}c. cu cls. I-VIII", Murga]i,
SIMONA CECILIA TUDORAN - C. N. "Carol I", Craiova, DENISA BISTRICEANU - C.
N. "Carol I", Craiova, VERONICA COTORA - C. N. "Carol I", Craiova, FLORENTINA
ZULEANU - }c. Nr. 9 "Petrache Poenaru", Craiova, VIRGINIA DOBRESCU - "}c. cu
cls. I-VIII", Desa, IRINA NUT{ - C. N. "}tefan Velovan", Craiova, MIHAELA RISTEA -
C. N. "Carol I", Craiova, IRINA PETRE - C. N. "Carol I", Craiova, GEORGIANA TOMA
- C. N. "Carol I", Craiova, TUDOR C{LIN - "}c. cu cls. I-VIII", D[buleni, ALEXANDRA
C{LIN - Lic. Teoretic "Constantin Br`ncoveanu", D[buleni, ADELA PAULA C{LIN -
Gr[dini\a Nr. 17, Craiova, FLORENTINA POPA - C. N. "Carol I", Craiova, LIANA
VELISCU - C. N. "Carol I", Craiova, GETA TEAU - C. N. "Carol I", Craiova, LIVIANA
G#RD - "}c. cu cls. I-VIII", Ocolna, MIHAELA MICU - "Lic. Teh. Al. Macedonski",
Meline]ti, Dolj, LIDIA EFTENIE - "}c. cu cls. I-VIII", Bor[scu, Gorj, ANDREEA STOICA
]i MIRCEA MARIO STOICA - Arad, AURICA ALDEA - "}c. cu cls. I-VIII Nr. 7",
Foc]ani, Vrancea, ELISABETA ALM{}AN - "}c. cu cls. I-VIII Nr. 3", Foc]ani, Vrancea,
EMILIA BATOG - "}c. cu cls. I-VIII Nr. 3", Foc]ani, Vrancea, MARIA COJANU - }c. cu
cls. I-VIII, Nr. 2 "Ion Basgan", Foc]ani, Vrancea, SILVIA CONDREA - }c. cu cls. I-VIII,
Nr. 10"Duiliu Zamfirescu", Foc]ani, Vrancea, PETRIA DOLIA - }c. cu cls. I-VIII, Nr.
10"Duiliu Zamfirescu", Foc]ani, Vrancea, R{DI|A DR#MBU - }c. cu cls. I-VIII, Nr. 2 "Ion
Basgan", Foc]ani, Vrancea, LUCIA VIORICA GRIGORE - "}c. cu cls. I-VIII Nr. 7",
Foc]ani, Vrancea, MARIANA ICHIM - }c. cu cls. I-VIII, Nr. 2 "Ion Basgan", Foc]ani,
Vrancea, GEORGETA IFRIM - "}c. cu cls. I-VIII Nr. 3", Foc]ani, Vrancea, TITINA MICU
- "}c. cu cls. I-VIII Nr. 3", Foc]ani, Vrancea, ANETA R{DULESCU - Liceul Ped. "Spiru
Haret", Foc]ani, Vrancea, MIRELA RUSU - }c. cu cls. I-VIII, Nr. 5 "Anghel Saligny",
Foc]ani, Vrancea, TUDORI|A STEREA - "}c. cu cls. I-VIII Nr. 3", Foc]ani, Vrancea,
LENU}A STOICA - }c. cu cls. I-VIII, Nr. 2 "Ion Basgan", Foc]ani, Vrancea, FELICIA
}TEFAN - "}c. cu cls. I-VIII Nr. 7", Foc]ani, Vrancea, ZOI|A TABAN - }c. cu cls. I-VIII,
Nr. 5 "Anghel Saligny", Foc]ani, Vrancea, GABRIELA T{B{CARU - }c. cu cls. I-VIII, Nr.
8 "Alexandru Vlahu\[", Foc]ani, Vrancea, NELA T{NASE - "}c. cu cls. I-VIII Nr. 3",
Foc]ani, Vrancea, MARI|ICA VI|{ - "}c. cu cls. I-VIII", Soveja, ELENA ZLOTA - }c. cu
cls. I-VIII, Nr. 9 "}tefan cel Mare", Foc]ani, Vrancea.

22 Revista de matematic[ din Craiova


*********************************************************************
* probleme teme *
*********************************************************************

GIMNAZIU

Clasa a V-a
Probleme selectate de: prof. Dorina Dr[cea, Craiova

T.G.4245. a) Afla\i x  N pentru care: 4x  5  2.


x7
b) Afla\i x  N pentru care: 1  1  1 .
3x  15 10x  5 4x  12
T.G.4246. Ar[ta\i c[ pentru orice num[r natural n, urm[toarele frac\ii sunt inducti-
bile: 2n  3 ; 2n  3 ; 5n  3 .
2n  2 2n  5 3n  2
T.G.4247. Determina\i n  N, pentru care: a) 7  N; b) 2n  5  N.
2n  1 n1
T.G.4248. Afla\i numerele naturale a ]i b astfel @nc`t:
(a  2 )(b  1 ) 13
a)  ; b) 12  3.
27 9 (a  3 )(b  1 ) 5
T.G.4249. Fie mul\imile:
A  a a  N, b  N  , a  2015; b  2015 .
b
 
B   ab ab este echivalent[ cu 3 . Afla\i cardA, B ]i cardB.
 cd cd 11 
T.G.4250. Simplifica\i frac\iile:
xy  yx
a) 3  6  9  . . .  3000 ; b) 21212121 ; c) 5x  5y .
5  10  15  . . .  5000 43434343
T.G.4251. Compara\i frac\iile:
b) a cu a  5 .
750 375
a) 2 125 ]i 3 250 ;
3 2 b b5
T.G.4252. C`te frac\ii cu numitorul 2 2  5 2  13 2 sunt cuprinse @ntre 11 ]i 12 ?
13 13
T.G.4253. Calcula\i: a) 1  1  1  . . .  1 .
2 6 12 2014  2015
b) 1  2  1  4  5  6  7  8  8  10 .
2 6 3 16 20 30 35 48 54 70
T.G.4254. Afla\i cifra x, ]tiind c[:
0, (1x )  0, (2x )  0, (3x )  . . .  0, (9x )  5, (54 ).

Editura Cardinal 23
T.G.4255. Scrie\i frac\ia 0, 2(3 ) ca o sum[ de 6 frac\ii zecimale periodice mixte.
T.G.4256. Calcula\i:
S  0, 1  2, 3  4, 5  6, 7  . . .  98, 99  100, 101  . . .  998, 999  . . .  2012, 2013.
T.G.4257. Efectua\i:
a) 0, 5 2 : 25  (0, 1  0, 8 )  : 0, 2; b) (2, 56 : 1, 6  0, 4 ) : 0, 4  12, 7 : 0, 5.
2 3

T.G.4258. Fie numerele


a  1  2  3  . . .  n ]i b  1  1  1  . . .  1 .
2 3 4 n1 2 3 4 n1
Afla\i media aritmetic[ a numerelor a ]i b.
T.G.4259. Compara\i numerele a ]i b, unde
a  1 1 1  1  ...  1 ]i b  1 1  1  1  . . .  1 .
2014 2 3 2014 2015 2 3 2015
T.G.4260. Afla\i x din egalit[\ile:
a) (14, 56  x )  0, 2  12; b) x : 2, 7  3, 1;
c) 2x  5, 16  x  3, 4 : 0, 5; d) x, (x )  0, (x )  0, 0(x )  10, 9.
T.G.4261. Afla\i suma primelor 2015 zecimale ale num[rului 4 .
7
T.G.4262. Latura unui p[trat este egal[ cu jum[tate din l[\imea unui dreptunghi, iar
perimetrul p[tratului este egal cu 0, 2 din perimetrul dreptunghiului. }tiind c[ aria
dreptunghiului este egal[ cu 100 m 2 , afla\i aria p[tratului.
T.G.4263. C`te dreptunghiuri au aria egal[ cu 18 cm 2 ]i au dimensiunile exprimate
prin numere naturale? Afla\i perimetrele lor.
T.G.4264. Un bazin are form[ de paralelipiped dreptunghic cu dimensiunile 10 m,
0, 8 dam ]i @n[l\imea egal[ cu 500 cm. a) C`\i litri de ap[ @ncap @n bazin?
b) La ce @n[l\ime se ridic[ apa @n bazin, dac[ @n bazin se pun 360.000 l ap[.
T.G.4265. Un acvariu are forma unui cub cu latura de 8 dm ]i este plin cu ap[. 30%
din ap[ se gole]te @ntr-un vas @n form[ de paralelipiped dreptunghic cu lungimea de
10 dm, l[\imea de 6 dm ]i @n[l\imea de 5 dm.
P`n[ la ce @n[l\ime se ridic[ apa @n fiecare vas?
T.G.4266. Dac[ mic]or[m latura unui p[trat cu 2, 5 m ]i alta cu 1, 5 m, ob\inem un
dreptunghi care are aria cu 70, 25 m 2 mai mic[ dec`t aria p[tratului.
Afla\i perimetrul p[tratului.
T.G.4267. Anda are v`rsta egal[ cu 0, (3 ) din v`rsta mamei, iar tat[l are v`rsta egal[
cu 1, (1 ) din v`rsta mamei. }tiind c[ cei trei au @mpreun[ 88 de ani, afla\i c`\i ani are
fiecare.
T.G.4268. 4 kg de mere ]i 3 kg de portocale cost[ 19, 3 lei, iar 2 kg de mere ]i 1 kg
de portocale cost[ 7, 9 lei. Afla\i c`t cost[ 1 kg de mere ]i c`t cost[ 1 kg de portocale.
T.G.4269. Din dou[ localit[\i pleac[ unul spre cel[lalt dou[ autovehicule cu vite-
zele de 60 km/h ]i respectiv 50 km/h. }tiind c[ ele se @nt`lnesc dup[ 45 de minute,
afla\i distan\a dintre cele 2 localit[\i.
#n c`t timp parcurge toat[ distan\a fiecare autovehicul?

24 Revista de matematic[ din Craiova


Clasa a VI-a
Probleme selectate de: prof. Constantin Cazacu, Craiova
prof. Sidonia Cazacu, Craiova
y
T.G.4270. Dac[ xy  z  xz , ar[ta\i c[: x  y  z.
T.G.4271. Se d[ xy  1 .
2
6x  y 2  yn  7  xn
a) Calcula\i ; b) Afla\i n  N ]tiind c[: xn  yn  N.
3x  2y
T.G.4272. Afla\i x din fiecare propor\ie de mai jos:
a) x  4  1 ; b) 2x  3  5 ;
x
c) 2  128 .
x2 7 3x  4 16 8 2 x2
T.G.4273. Determina\i numerele x, y, z ]tiind c[ x  y, y  z, x  z sunt direct pro-
por\ionale cu 10, 12 ]i respectiv 14, iar x  y  z  24000.
y
T.G.4274. Determina\i numerele naturale x, y, z ]tiind c[: x   11 z .
4 3
T.G.4275. O echip[ de 15 muncitori ar termina o lucrare @n 20 de zile. Dup[ 4 zile
de lucru mai vin 5 muncitori: Cu c`te zile se termin[ mai repede lucrarea @n aceast[
situa\ie?
T.G.4276. O echip[ de 12 muncitori poate construi 7 m de autostrad[ dac[ lucreaz[
14 zile c`te 3 ore pe zi. De c`\i muncitori este nevoie pentru a construi 9 m de
autostrad[ dac[ lucreaz[ 6 zile c`te 6 ore pe zi?
T.G.4277. O cantitate de ro]ii este recoltat[ de 24 de muncitori care recolteaz[
fiecare c`te 40 kg pe or[ @n 3 zile lucr`nd c`te 9 ore pe zi. De c`\i muncitori este
nevoie pentru a recolta aceea]i cantitate de ro]ii dac[ fiecare muncitor recolteaz[ c`te
36 kg de ro]ii pe or[ lucr`nd 4 zile c`te 6 ore pe zi?
T.G.4278. #ntr-o urn[ sunt 2 bile albe ]i 3 bile negre. Se extrag simultan dou[ bile.
a) Care este probabilitatea ca ambele bile s[ fie albe?
b) Care este probabilitatea ca bilele s[ aib[ culori diferite?
T.G.4279. Dac[ a, b, c sunt numere ra\inale pozitive astfel @nc`t a  3 ]i bc  1
b 4
compara\i a ]i c.
T.G.4280. Dup[ trei cre]teri consecutive cu 10% un produs cost[ 67881 lei. Care a
fost pre\ul ini\ial al produsului?
T.G.4281. Pre\ul unui obiect cre]te cu 60%. Cu ce procent ar trebui mic]orat pre-
\ul, pentru a ajunge la pre\ul ini\ial?
T.G.4282. Calcula\i:
a) 2  23  (3  4 ) ; b) 2  4 2  2  (3 ) ;
10 2

c) 5  32  22  (4)  (52 ); d) 2  32  3 2  (3)  (5 2 ).


2 2

Editura Cardinal 25
T.G.4283. Rezolva\i @n Z ecua\iile:
a) 2x  3  1; b) 2x  3  4  1; c) 2x  3  4  3  1.
T.G.4284. Calcula\i: a) (15 )  (2 )  6  (4  6  5  1 );
b) (5 )  (2 )  (3 )  (2 )  6  (2 )  (2 2 );
c) (4 ) : (8  4 )  (12 )  (10 ) : (14  8 );
d) (3 ) : 3 4  (3 2 )  (2 4 )  (2 ) : (2 5 );
5 2

e) (27 )  (9 ) : (81 ) .


25 15 25

T.G.4285. Rezolva\i @n Z ecua\iile:


a) 3(x  5 )  3  2(x  1 ); b) 3 x  2  7 2  x  16;
c) x  1  3(x  1 )  5  3x; d) (x  1 )  8.
2

T.G.4286. Se consider[ triunghiul isoscel ABC cu AB  AC ]i m(BAC )  120  .


Se construiesc @n exteriorul triunghiului ABC, triunghiurile echilaterale ABE ]i ACF.
a) Ar[ta\i c[: EF  BC . b) Afla\i m(EFB ).
T.G.4287. Prin v`rfurile A ]i C ale triunghiului isoscel ABC cu AB  BC se duc
paralele la bisectoarea unghiului B care intersecteaz[ dreptele AB ]i BC @n punctul M ]i
respectiv F. Ar[ta\i c[ triunghiul BEF este isoscel.

T.G.4288. #n triunghiul ABC, AB  1 ]i m(A )  60  . Afla\i m[surile unghiurilor


AC 2
triunghiului ABC.
T.G.4289. #n triunghiul isoscel ABC cu baza BC punctul H este ortocentrul triun-
ghiului. Dac[ N este simetricul lui H fa\[ de AC ]i AN  NC ar[ta\i c[ triunghiul
ABH este isoscel.
T.G.4290. Triunghiul ABC are m(A )  90  ]i @n[l\imea AD are lungimea de 6 cm
(D  BC ). Dac[ N ]i P sunt simetricele punctului D fa\[ de AB respectiv AC, cal-
cula\i aria triunghiului DNP.
T.G.4291. #n triunghiul isoscel ABC cu AB  AC, BD  AC, D  AC ]i BE este
bisectoarea unghiului ABC, E  AC. Dac[ m(EBD )  30  , afla\i m[surile unghiu-
rilor triunghiului ABC.
T.G.4292. Triunghiul ABC cu AB  AC are BC  AB. Se consider[ punctul D pe
latura BC astfel @nc`t CD  AB  BC ]i C  BD. Dac[ m(BAC )  40  afla\i
m(ADB ).
T.G.4293. #n triunghiul ABC se consider[ punctele D  AB ]i E  AC astfel @nc`t
AD  AE . Dac[ BE  CD  F ]i AF este bisectoarea unghiului A ar[ta\i c[
triunghiul ABC este isoscel.
T.G.4294. M[surile unghiurilor A, B ]i C ale triunghiului ABC sunt direct
propor\ionale cu numerele 10, 2 ]i respectiv 6. Ar[ta\i c[ exist[ un punct M  AB
astfel @nc`t P AMC  AB  AM.

26 Revista de matematic[ din Craiova


Clasa a VII-a
Probleme selectate de: prof. Cristiana Seinu, Craiova

T.G.4295. Calcula\i: a) 7  4 3  4  2 3 ;

b) 7  2 10  14  6 5  6  4 2 .

17  4 17  4
T.G.4296. Ar[ta\i c[  este num[r @ntreg.
4  17 17  4
T.G.4297. Ar[ta\i c[ numerele a ]i b sunt naturale unde
a  9  80  5 , b  11  6 2  3  2 2 .

T.G.4298. Fie A  1  1  ...  1 , unde n  N, n  2.


1 2 2  3 n1  n
Determina\i valorile lui n, n  100 pentru care A este ra\ional.
T.G.4299. Determina\i x, y  Q @n urm[toarele condi\ii:
a) (x  2 ) 2  y  2 ; b) x(1  2 )  3  y  2 2 .
2

T.G.4300. Compara\i numerele reale x ]i y @n cazurile urm[toare:


a) x  n  3  n y  n  2  n  1 , n  N .
b) x  n  6  n  2 y  n  1  n  7 , n  N.
T.G.4301. Demonstra\i c[ urm[toarele numere sunt ira\ionale:
a  1  6 2015 , b  1  2  3  . . .  14 .
T.G.4302. a) Determina\i numerele reale a, dac[ a  2 ]i 2  a 2 sunt ambele ra\i-
onale.
b) Determina\i num[rul real x pentru care numerele x  2 ]i x 2 sunt
ambele ra\ionale.
T.G.4303. Determina\i mul\imile
A  x  Z 2x  1  Z , B  x  Z 3x  5  Z .
1  3x x1
T.G.4304. Determina\i tripletele de numere reale (x, y, z ) care @ndeplinesc simultan
xy yz
condi\iile: a) xyz  2, b) z  2, c) x  2 2 , d) xz
y  2 sunt numere naturale.
3

T.G.4305. Ar[ta\i c[ dac[ x, y sunt numere reale, atunci (2x  3 )  (3y  5 ) 


2 2

 4x 2  9y 2 , dac[ ]i numai dac[ 6x  15y  17.

T.G.4306. Demonstra\i c[ a  b  2 a  b , oricare ar fi numerele reale pozi-


2
tive a ]i b. Ar[ta\i c[: 1994  1995  1996  1997  1998  5 1996 .

Editura Cardinal 27
T.G.4307. Demonstra\i c[:
x 2  4x  13  4x 4  8x 2  29  9x 6  12x 3  53  15.
Pentru ce valori ale lui x putem avea egalitate?
T.G.4308. Rezolva\i @n Z ecua\iile:
a) 2x  3y  1; b) 1  1  1 ;
4x 5y 20
c) 1  1  1 ; d) 3xy  9x  2y  13.
2x 3y 4
T.G.4309. Descompune\i @n factori:
a) 2x 2  5x  3; b) a 4  2a 3  2a 2  2a  1; c) x 4  x 2  1.
T.G.4310. Dac[ a, b, c sunt lungimile laturilor unui triunghi ]i p este semiperimetrul
p3
s[u, atunci exist[ inegalitatea: abc(p  a )(p  b )(p  c )  .
8
T.G.4311. Fie ABCD paralelogram. Fie E simetricul lui A fa\[ de D. Dac[
AC  BD  O ]i AC  BE  M, ar[ta\i c[:
a) BCDE este paralelogram, b) OM  1 AC.
6
T.G.4312. #n patrulaterul ABCD avem AB  CD ]i AC  BD  O. Paralela prin O
la AB intersecteaz[ AD ]i BC @n punctele P, respectiv Q. Dac[ PQ  AB  CD , de-
2
monstra\i c[ ABCD este paralelogram.
T.G.4313. Fie triunghiul ABC dreptunghic m A  90  ]i AD  BC, D  BC.

Demonstra\i c[: (1) AD  BC  AB  AC. (2) 1  1  1 .


AD 2 AB 2 AC 2
T.G.4314. Fie ABCD trapez, AB  CD ]i AC  BD  O.
Ducem prin O, MN  AD, unde M  AB, N  CD ]i PQ  BC, P  AB ]i Q  CD.
Ar[ta\i c[: AM  DN  AP  DQ.
T.G.4315. Se consider[ triunghiul ABC cu D  (BC ) ]i P  (AD ) astfel @nc`t triun-
ghiul APB ]i triunghiul ACP sunt echivalente.
a) Demonstra\i c[ AD este median[.
b) Determina\i pozi\ia lui P  (AD ) astfel @nc`t: 2  A ABC  3  A PBC.
T.G.4316. Fie triunghiul ABC cu m B  2  m C . Construim bisectoarea BE a
unghiului ABC, E  (AC ). Paralela prin C la BE taie dreapta AB @n punctul D.
Demonstra\i c[: a) AB 2  AC  AE;
b) ABC  ACD;
c) AE 2  EB 2  2  AE  EB  AB 2  AB  BD;
d) bisectoarele unghiurilor BEC ]i DBC sunt concurente @n punctul
O, centrul cercului circumscris triunghiului BCD.
T.G.4317. Fie ABCD patrulater inscriptibil, care nu este trapez. Dac[ AB  CD 
 P, ar[ta\i c[ PA  PB  PC  PD. (Acest produs se nume]te puterea punctului P fa\[
de cercul circumscris patrulaterului ABCD ).

28 Revista de matematic[ din Craiova


T.G.4318. Se consider[ cercul de centru O tangent la dreapta d @n punctul T. O
paralel[ la OT intersecteaz[ cercul ]i dreapta d @n punctele A, B respectiv C. (B este
@ntre A ]i C).Fie H punctul diametral opus lui B, iar HT intersecteaz[ pe AB @n N.
a) Ar[ta\i c[ triunghiul HBN este isoscel ]i TC  1 HA.
2
b) Dac[ E este mijlocul lui (BN ), ar[ta\i c[ AOTE este inscriptibil.
T.G.4319. Fie punctul D interior triunghiului ABC cu AB  AC ]i m A  100 
astfel @nc`t m DBC  30  ]i m DCB  20  . Afla\i m BAD .

Clasa a VIII-a
Probleme selectate de: prof. Constantin Basarab, Craiova

T.G.4320. Determina\i m  R cu proprietatea c[ punctele A(1; 1 ), B(2; 7 ) ]i


Cm  1; 2m  1 sunt coliniare.
T.G.4321. f : R  R, f (x )  (2  5 )x  5 .
a) Ar[ta\i c[ A(1; 2 )  G f .
b) Rezova\i @n R inecua\ia f (x )  2  0.
c) Determina\i numerele ra\ionale a ]i b, ]tiind c[ M(a; b  b 5 ) se afl[ pe
graficul lui f.
T.G.4322. Determina\i func\iile liniare al c[ror grafice trec prin A(1; 1 ) ]i formea-
z[ cu axa absciselor un unghi de 45  .
T.G.4323. Fie func\iile f : RR, f (x)  2x  7 ]i g : RR, g(x )  x  3.
a) Reprezenta\i grafic f ]i g @n acela]i sistem de coordonate.
b) Determina\i perimetrul ]i aria triunghiului format de grafice ]i axa ordo-
natelor.
T.G.4324. Fie f : R  R, f (x )   1 x  3 ]i g : R  R, g(x )  2x  1.
2
a) Reprezenta\i grafic f ]i g @n acela]i sistem de coordonate.
b) Determina\i m[sura unghiului dintre grafice.
c) Calcula\i suma S  f (1 )  f (2 )  f (3 )  f (4 )  . . .  f (19 )  f (20 ).
T.G.4325. Rezolva\i @n R  R sistemul de ecua\ii:
 2x  y  6 x  2y  6
 2x  y  1  x  2y  1  4
 2x  y  4 x  2y  8 .
  0
 2x  y  1 x  2y  1
T.G.4326. Rezolva\i @n R ecua\ia: 2 1  2 1  1  3 .
x  x x  3x  2 x 2  5x  6 x  3
T.G.4327. Rezolva\i @n R ecua\iile:
a) max(2x  3; x  2 )  3x  1; b) min(x  2; 3  x )  2x  1.

Editura Cardinal 29
T.G.4328. Rezolva\i @n R ecua\iile:
a) x  3  5  x  x  4; b) x  3  9  3x  x 2  9.

T.G.4329. Rezolva\i @n R inecua\iile: a) x  2  x  3  2;


2 3
b) x 2
 2x  2  0; c) 2 x  1  x 5  5 .
( )
2x  3
T.G.4330. Fie E(x )  ax 2  bx  c, unde a, b, c  R.
a) Pentru a  3, b  4 ]i c  1 rezolva\i @n R ecua\ia E(x )  0.
b) Pentru a  b  1 ]i c  1 rezolva\i @n R ecua\ia:
E(x )  x 2  E(x )  x  0.
c) Pentru a  b  a ]i c  5 afla\i valoarea minim[ a lui E(x ) c`nd x  R.
T.G.4331. Fie ecua\ia x2  2(m  1) x  m2  m  1  0, m  R.
a) Pentru m  2, rezolva\i ecua\ia.
b) Determina\i m, ]tiind c[ ecua\ia are solu\ia m.
c) Determina\i m pentru care ecua\ia are dou[ solu\ii diferite.
T.G.4332. }tiind c[ 3x 2  5xy  2y 2  0 ]i y  0, determina\i xy .

T.G.4333. Fie E(x )  3x


3
 18x 2  27x , unde x  N  3.
(x  x ) (x 2  x  6 )
2

a) Rezolva\i @n Z ecua\ia x 2  x  6  0.
b) Ar[ta\i c[ E(x ) se simplific[ cu x 2  3x.
c) Determina\i n  N pentru care E(n ) se simplific[ cu 6.
T.G.4334. Un c[l[tor a parcurs @n prima zi 1 din tot drumul ]i @nc[ 6 km, a doua zi
4
2 din rest ]i @nc[ 8 km, iar a treia zi 5 din rest ]i @nc[ 2 km. }tiind c[ i-au r[mas de
7 13
parcurs 30 km, afla\i lungimea drumului ]i c`t a parcurs @n fiecare zi.
T.G.4335. ABCDA  B  C  D  este prism[ patrulater[ regulat[ cu AB  4 cm ]i
AA   4 3 cm. Se cere:
a) aria total[, volumul ]i diagonala prismei;
b) sinusul unghiului dintre (A  BD ) ]i (C  BD );
c) unghiul dintre A  D ]i BC  ;
d) distan\a de la D  la (A  BD ).
T.G.4336. Se d[ prisma triunghiular[ regulat[ ABCA  B  C  cu AB  8 cm ]i AA  
 6 cm. S[ se afle:
a) aria total[ ]i volumul prismei;
b) distan\a de la B  la AC;
c) cosinusul unghiului dintre A  C ]i BC  .
T.G.4337. Un vas @n form[ de piramid[ patrulater[ regulat[ are latura bazel 12 cm ]i
@n[l\imea 20 cm. Vasul este plin cu ap[ p`n[ la mijlocul @n[l\imii. Care va fi cubul
@n[l\imii p`n[ la care se ridic[ apa dac[ vasul se @ntoarce cu v`rful @n jos?

30 Revista de matematic[ din Craiova


T.G.4338. ABCA  B  C  este o prism[ triunghiular[ regulat[ cu AB  8 cm, A  B  10
cm, D este mijlocul lui A  B ]i E mijlocul lui BC. Se cere:
a) aria total[ ]i volumul prismei;
b) aria triunghiului DBC;
c) aria patrulaterului A  DEC.
T.G.4339. ABCDA  B  C  D  este un cub cu AB  4 cm, M este mijlocul lui A  D   ]i N
mijlocul lui D  C  . Se cere:
a) aria lateral[ ]i volumul cubului;
b) aria patrulaterului MNCA;
c) sinusul unghiului dintre AM ]i DN.
T.G.4340. VABCD este piramid[ patrulater[ regulat[ cu toate muchiile egale cu 12
cm. Fie punctele P  (AB ) ]i Q  (BC ), astfel @nc`t AP  CQ  1 AB. Se cere:
3
a) aria total[ ]i volumul piramidei;
b) Determina\i pozi\ia punctului E pe VO (unde O este centrul bazei), ]tiind
c[ m QEP  90  ;
c) Dac[ m QEP  90  , afla\i m[sura unghiului dintre (AEC ) ]i (QEP ).
T.G.4341. #ntr-o piramid[ triunghiular[ regulat[ muchia lateral[ formeaz[ cu planul
bazei un unghi de 45  . }tiind c[ latura bazei este 12 cm, se cere:
a) aria total[ ]i volumul piramidei;
b) distan\a de la un v`rf al bazei la fa\a opus[;
c) se sec\ioneaz[ piramida cu un plan paralel cu baza, astfel @nc`t aria
lateral[ a trunchiului ob\inut este dublul ariei laterale a piramidei mici formate. Afla\i
volumul trunchiului.
T.G.4342. Un trunchi de con circular drept are sec\iunea axial[ un trapez isoscel cu
bazele de 10 cm ]i 4 cm ]i diagonalele perpendiculare. Se cere:
a) aria lateral[ ]i volumul trunchiului de con;
b) aria lateral[ ]i volumul conului din care provine trunchiul de con.
T.G.4343. Un con circular drept este circumscris unei sfere cu raza 2 3 cm. }tiind
c[ @n[l\imea conului este 6 3 cm, se cere:
a) aria total[ ]i volumul conului;
b) aria ]i volumul sferei.
T.G.4344. ABCDA  B  C  D  este un trunchi de piramid[ patrulater[ regulat[ cu
volumul 378 cm 3 , @n[l\imea 6 cm ]i latura bazei mari 12 cm. Se cere:
a) Latura bazei mici A  B  .
b) Dac[ A  B   3 cm, calcula\i aria lateral[ a trunchiului.
c) #n[l\imea piramidei din care provine triunchiul.
d) Dac[ V este v`rful piramidei, O centrul bazei mari a triunchiului ]i T
proiec\ia lui O pe (VAB ), afla\i aria triunghiului VTA.

Editura Cardinal 31
LICEU
Clasa a IX-a
Probleme selectate de: prof. C[t[lin Cristea, Craiova

T.L.3659. Fie func\iile f, g : RR, pentru care (f  g )(x)  3x  1.


a) Calcula\i f (5 ) ]tiind c[ g(4 )  5.
b) Determina\i g(x ), ]tiind c[ f (x )  x  2014.
c) Determina\i f (x ), ]tiind c[ g(x )  x  2015.
T.L.3660. Se consider[ func\iile f, g : RR, f (x)  ax  b, g(x )  bx  a, unde a ]i
b sunt numere reale. S[ se arate c[ dac[ f (1 )  g(1 ), atunci f  g.
T.L.3661. Se consider[ func\ia f : R  R, f (x )  2x  5.
a) Reprezenta\i grafic func\ia f.
b) Determina\i imaginea intervalului 1, 3 prin func\ia f.
c) Rezolva\i @n R ecua\ia: (f  f )(3x )  1  2x.
x  x2 f (x 1 )  f (x 2 ) ( )
d) Ar[ta\i c[: f 1  ,  x 1 , x 2  R.
2 2
T.L.3662. Rezolva\i @n R inecua\iile: a) x  1 x  1  0;
2
2(x  1 )  (x  1 )  (x  1 )(x  1 )
2 2
b) 5  3x  0 ; c)  0.
16  8x  2x 2 (x  1 ) 2  (x  1 ) 2

T.L.3663. Determina\i m  R astfel @nc`t: x 2 2x  1  m  0, ( ) x  R.


2

x  3x  3
T.L.3664. Fie func\ia f : R  R, f (x )  x 2  ax  1, a  R.
a) Determina\i a ]tiind c[ dreapta de ecua\ie x  1 este ax[ de simetrie.
b) Determina\i a ]tiind c[ graficul func\iei f este tangent axei Ox.
c) Determina\i a astfel @nc`t lm f  3, .
T.L.3665. Determina\i cea mai mic[ ]i cea mai mare valoare pentru func\ia
f : 1, 5  R, f (x )  2x 2  x  10.
T.L.3666. Determina\i func\ia de gradul al doilea care trece prin punctul A(1, 1 ) ]i
care are v`rful @n punctul V  1 , 3 .
2 4
T.L.3667. a) Determina\i a  R pentru care parabola y  x 2  2x  a  1 ]i dreapta
y  2x  3 au dou[ puncte distincte comune.
b) Determina\i coordonatele punctelor de intersec\ie dintre parabola de
ecua\ie y  x 2  x  1 ]i dreapta de ecua\ie y  2x  1.
T.L.3668. Se consider[ func\ia f : R  R, f (x )  x 2  2x  m, m parametru real.
a) Determina\i m  R astfel @nc`t: f (x )  0, ( ) x  0.
b) Verifica\i c[: f ( 2015  1)  f ( 2015  1), () m  R.

32 Revista de matematic[ din Craiova


T.L.3669. a) Determina\i valorile reale ale parametrului m ]tiind c[ solu\iile reale
ale acua\iei x 2  (m  1 )x  3  0 verific[ rela\ia x 1  3x 2 .
b) Ar[ta\i c[ solu\iile x 1 ]i x 2 ale ecua\iei x 2  2mx  m 2  1  0 verific[
rela\ia x 1 x2  (x1  x 2 )  2  0, pentru orice m  R.
T.L.3670. Ar[ta\i c[ oricare ar fi m  R, parabola asociat[ func\iei f : R R, f (x) 
 x 2  2x  m 2  2 este situat[ sub axa Ox.
T.L.3671. Dac[ x 1 ]i x 2 sunt r[d[cinile ecua\iei x 2  x  3  0 calcula\i:
a) x 21  x 22 ; b) x 31  x 32 ; c) x 21  x 32 .
T.L.3672. Fie func\ia f : R R, f (x )  x 2  4x  m, unde m este un parametru real.
a) Determina\i valorile lui m pentru care: f (x )  0, () x  R.
b) Determina\i minimul func\iei f.
c) Pentru m  0, afla\i valorile reale ale lui x pentru care (f  f )(x )  12.
T.L.3673. Rezolva\i @n R inecua\iile:
x(x  2 ) x(x  2 ) x(x  1 )
a)  x  6; b) 2  1; c) x  1.
x1 x 1 x1 2
 
T.L.3674. S[ se determine func\iile f : N  N cu proprietatea:
f 2 (n ) f (n  1 )  1 ( )
 n  ,  n  1.
f (n )  n f (n ) 2
T.L.3675. Demonstra\i egalit[\ile:

a) tg15 0  sin 10 0  sin 20 0 ;


0 0
b) 16  sin 20 0  sin 40 0  sin 60 0  sin 80 0  3.
cos 10  cos 20
T.L.3676. Calcula\i: a) cos 10 0  cos 20 0  cos 30 0  . . .  cos 170 0 .
b) sin 1   sin 2   . . .  sin 358   sin 359  .

T.L.3677. Demonstra\i c[: tg 2   7 .


8 40
T.L.3678. Demonstra\i egalitatea: tga  ctga  1  cos2 a  12 .
2

1  sin a cos a
T.L.3679. Fie    ,  , astfel @nc`t cos 2  1 . Calcula\i cos  ]i cos 3.
2 2
T.L.3680. Demonstra\i c[: sin   sin 3  sin 5  1 .
14 14 14 8
T.L.3681. Dac[ 1  ctg   x  2 , x, y  0,  
4 1  ctg y 2 , ar[ta\i c[ x  y  2 .
T.L.3682. a) Fie triunghiul ABC cu AB  8, AC  8 2 , BC  8 3 . Calcula\i raza
cercului circumscris triunghiului.
b) Demonstra\i c[ @ntr-un triunghi oarecare ABC are loc egalitatea:
sin 2 A  sin 2 B  sin 2 C  1  2 sin A sin B cos C.
T.L.3683. Ar[ta\i c[ triunghiul ABC @n care a  2b cos C este isoscel.

Editura Cardinal 33
Clasa a X-a
Probleme selectate de: prof. Raluca Ciurcea, Craiova

T.L.3684. Afla\i valorile lui x pentru care au sens expresiile:


a) arcsin(x 2  5x  5 ); b) arccos 4 x  1 ; c) arctg 6 x 2  2x  3 ;
T.L.3685. Demonstra\i inegalit[\ile:
b) arctg 1  arctg 3   .
2 6 1 ;
5  arcsin 5  2
a) arcsin
4 5 4
T.L.3686. Rezolva\i ecua\iile:
2
a) arccos(x 2  4x  4 )  ; b) arcsin x  arccos x  5  0.
18
T.L.3687. Rezolva\i ecua\iile: a) 5x 2  1  x  3;
b) 3 x  7  x  1 ; c) x  3  x  4  7;
8
d) 3 2x  1  3 2x  1  2; e) 5 x  1  2 5 x 2  2x  1  15.

T.L.3688. Rezolva\i ecua\iile: a) (0, 4 )


x 2 2x3
2
5;
b) 5 2x  3  5 x  10; c) 4 x1
2 x1
 6  0;
4 4
d) 5 x  12 x  13 x ; e) (8  3 7 )  (8  3 7 )  16.
x2 x2

T.L.3689. Afla\i valorile parametrului real m pentru care ecua\ia


9 x  4  3 x  m  3  0 are dou[ r[d[cini reale.
T.L.3690. Rezolva\i ecua\iile: a) log 3 (x  2 )  log 3 (3x  24 )  4;

b) lg 2 x  lg x  30  0; c) log x x 2  20  1 ;
2
d) x 5 log 3 x1  729; e) 3 x  2 log 4 x  10.
a) sin 2x    
3
T.L.3691. Rezolva\i ecua\iile trigonometrice: ;
5 2
b) cos(4x )  7 ; c) tg 5x     3 ; d) ctg(2x   )  1;
9 6
e) sin(3x   )  sin(x ); f) tg x    ctg 2x  2  0.
3 3
T.L.3692. Rezolva\i ecua\iile: a) sin x  3 cos x  1;
b) sin(2x )  4 cos(2x )  5; c) 3 sin(x   )  4 cos(x   )  2, 5.
T.L.3693. Determina\i num[rul func\iilor f : A  B ]tiind c[ A are 5 elemente ]i
B  x  Z  a  R astfel @nc`t x  sin a.
T.L.3694. Dac[ A, B R astfel @nc`t card(A  B )  12, card(A  B )  2 ]i 3cardA 
 4cardB determina\i c`te func\ii f : B  A sunt:
a) injective; b) strict cresc[toare; c) strict monotone.

34 Revista de matematic[ din Craiova


T.L.3695. a) Afla\i c`te numere naturale de patru cifre exist[ cu proprietatea c[
toate cifrele acestora sunt pare.
b) Afla\i c`te numere naturale de patru cifre distincte exist[, av`nd
proprietatea c[ toate cifrele acestora sunt pare.
c) Afla\i c`te numere naturale de patru cifre distincte exist[, av`nd
proprietatea c[ toate cifrele acestora sunt impare.
T.L.3696. Afla\i num[rul termenilor ira\ionale din dezvolt[rile:
a) ( 4 3  2 ) ; b) ( 3 4  2 ) .
100 2015

2015
T.L.3697. Determina\i rangul celui mai mare termen al dezvolt[rii 2  3 .
5 5
T.L.3698. Afla\i rangul termenului care con\ine:
2015
a) pe x 3 @n dezvoltarea (x 2  2x ) ; b) pe x 0 @n dezvoltarea x 3  12
0 21
.
x
T.L.3699. Calcula\i: a) S  C 2015  C 2015  C 2015  . . .  C 2015;
0 2 4 2014

b) T  C 12015  C 42015  C 72015  . . .  C 2014


2015;

c) U  C 77  C 78  C 79  . . .  C 72015.

T.L.3700. Demonstra\i c[ urm[toarele numere sunt @ntregi:


a) 2015! ; b) 8! ; c) 12!3 ; d) 37! .
1977!  38! 4!  4! (4! ) 5!  7!  10!  15!
T.L.3701. }tiind c[ num[rul elevilor unei clase este cuprins @n intervalul (25; 50 ) ]i
exact 15% dintre ace]tia au ob\inut nota 10 la matematic[, afla\i c`\i elevi sunt @n clas[
]i c`\i au nota 10 la matematic[.
T.L.3702. Se plaseaz[ o sum[ de 3000 u. M. Pe o durat[ de 7 ani @n regim de
dob`nd[ compus[ cu rata dob`nzii de 5%. Care este capitalul final?
T.L.3703. Trei sportivi trag c`te un foc asupra unei \inte, probabilit[\ile de a o
nimeri fiind respectiv 1 ; 5 ]i 3 . Care este probabilitatea ca \inta s[ fie nimerit[ exact
2 6 8
de 2 ori?
T.L.3704. Care este probabilitatea de a ob\ine un multiplu de 3 sau un multiplu de 7
dac[ alegem la @nt`mplare un num[r natural de trei cifre?
T.L.3705. Mijloacele laturilor AB, BC, CA ale triunghiului ABC sunt punctele
D 1, 2 ), E(2, 3 ), F(0, 5 ). Afla\i:
(
a) coordonatele v`rfurilor triunghiului ABC;
b) lungimile medianelor triunghiului ABC .
T.L.3706. Consider[m triunghiul ABC, cu A(3, 0 ); B(0, 4 ) ]i C(1, 3 ).
a) Afla\i ecua\ia dreptei BC.
b) Afla\i ecua\ia dreptei suport a @n[l\imii din A a triunghiului ABC.
c) Afla\i lungimea @n[l\imii din A a triunghiului ABC.
T.L.3707. Afla\i valorile parametrului real m pentru care punctele
A(3  m, 2 ), B(1, 4 ), C(2, m ) sunt coliniare.
T.L.3708. Determina\i aria patrulaterului ABCD, unde A(5, 0 ), B(1, 2 ), C(3, 1 ),
D(1, 4 ).

Editura Cardinal 35
Clasa a XI-a
Probleme selectate de: prof. Felician Preda, Craiova

cos t  sin t
T.L.3709. Fie A  , t  R. Calcula\i A n , pentru orice n  N.
sin t cos t
T.L.3710. Studia\i compatibilitatea urm[toarelor sisteme folosind teorema Krone-
cker - Capelli:
 x  2y  z  2
 3x  y  4  3x  5y  1 
a)  ; b)  ; c)  2x  y  3z  9 .
 6x  2y  1  2x  y  2  3x  y  2z  1

T.L.3711. Rezolva\i prin metoda lui Cramer urm[toarele sisteme de ecua\ii liniare:
 2x  y  5z  7  2x  3y  2z  0
 3x  y  3  
a)  ; b)  x  4y  z  0 ; c)  x  4y  6z  0 .
 2x  5y  19  4x  3y  2z  2  3x  y  10z  0
 
T.L.3712. Afla\i a  R, ]tiind c[ urm[toarele sisteme sunt compatibil determinate:
 x  ay  2z  1  a  ax  y  z  2
 
a)  x  (1  2a )y  3z  2  2a ; b)  x  ay  z  1 .
 x  a y  (a  3 ) z  a  1  x  y  az  3
 
T.L.3713. Se consider[ numerele reale a, b, c, f : RR, f (x)  x3  2x  3 ]i determi-
1 1 1 1 1 1
nan\ii A  a b c ]i B  a b c .
a3 b3 c3 f (a ) f (b ) f (c )
a) S[ se arate c[ A  (a  b ) (b  c ) (c  a ) (a  b  c ).
b) S[ se arate c[ A  B.
 ax  by  cz  1

T.L.3714. Fie sistemul  cx  ay  bz  1 unde a, b, c  R ]i a  b  c  0.
 bx  cy  az  1

Ar[ta\i c[ sistemul este incompatibil.
 x  y  mz  1

T.L.3715. Se consider[ sistemul de ecua\ii liniare  mx  y  mz  1  m .
 mx  3y  3z  1

Afla\i m  R pentru care sistemul este incompatibil.
T.L.3716. Fie triunghiul ABC, cu laturile AB  c, BC  a, CA  b ]i sistemul
 ay  bx  c

 cx  az  b . S[ se demonstreze c[ pentru orice tri-
 bz  cy  a

unghi sistemul are solu\ie unic[.

36 Revista de matematic[ din Craiova


T.L.3717. Fie m  R ]i dreptele d 1 : x  2y  3, d 2 : 3x  4y  1, d 3 : 4x  3y  m.
a) S[ se determine m astfel @nc`t dreptele s[ fie concurente.
b) S[ se demonstreze c[ exist[ o infinitate de valori ale lui m pentru care
v`rfurile triunghiului determinat de cele trei drepte au toate coordonatele @ntregi.
1 1 1
T.L.3718. Fie B  0 1 1  M 3 ( R ).
0 0 1
a) Ar[ta\i c[ B este inversabil[.
b) Calcula\i B1 ]i Bn , n  N .
a b
T.L.3719. Fie matricea X  , a, b  R.
1 1
i) S[ se calculeze X 2 .
ii) S[ se arate c[ X este inversabil[ @n M 2 (R )  a  b  0.
iii) Afla\i a ]i b pentru care X 3  O 3 .
T.L.3720. C`te matrice X  M 2 (R ) exist[ astfel @nc`t X 2  X?
x 1
T.L.3721. S[ se calculeze lim 2 .
x1 x  1
T.L.3722. S[ se determine m  R, astfel @nc`t x lim x 2  ax  x  5.
T.L.3723. Fie f : D  R, f (x )  x 2  1 , unde D R. Afla\i domeniul maxim de
defini\ie al func\iei ]i asimptotele func\iei pe domeniul maxim.
T.L.3724. Afla\i punctele de extrem ale func\iei f : RR, f (x)  (x2  x  1) ex .
T.L.3725. Prelungi\i prin continuitate urm[toarele func\ii:
 2x 3  x  1, x  (; 1 )
a) f (x )   .
 2x, x  (1;  )
b) f (x )  x n sin 1x , pentru x  0, unde n  N 
T.L.3726. Studia\i continuitatea func\iilor:
 3x 2  5 x , x  Q  sin x, x  RQ
f : R  R, f (x )   2 , g : R  R, g(x )   .
 x  3x, x  RQ  cos x, x  Q
T.L.3727. Demonstra\i c[ urm[toarele func\ii au proprietatea lui Darboux.
 x  2, x  1 
 e  1, x  0
x
f (x )   x1 , g(x )   2 sin x .
 e  2, x  1 
 x ,x0
T.L.3728. Care dintre urm[toarele func\ii au proprietatea lui Darboux:
 2015, x  0  x  2, x  5
a) f (x )   ; b) f (x )   ;
 2016, x  0  x  3, x  5
 sin 1 , x  0
 x 
 1  sin 1x , x  0
c) f x   3 , x  0 ;
( ) d) f (x )   .

 1  2, x  0

 cos x , x  0

Editura Cardinal 37
T.L.3729. Fie func\ia f : (; 1 )  (0,  )  R, f (x )  ln 1  1x .
a) Studia\i monotonia func\iei;
b) Stabili\i asimptotele func\iei;
f (1 )  . . .  f (n ) ( )
c) Fie ]irul (a n ) n N cu a n  n ,  n  N. S[ se calculeze lim a .
n n
T.L.3730. S[ se demonstreze inegalit[\ile:
i) sin a  sin b  a  b , () a, b  R.
ii) a 2b  ctgb  ctga  a 2b , () a, b  Rk  k  Z.
sin a sin b 2
T.L.3731. Demonstra\i inegalit[\ile: a) e x  1  x  x , ( ) x  0;
1! 2!
b) e  1  ln x,  x  0;
x1 ( ) c) sin x  x, ( ) x  0;
d) arctgx  x, ( ) x  0; e) ln x  x 1 , ( ) x  0.
T.L.3732. Fie a1 , a2 , . . ., an  R , astfel @nc`t a1  ax2  . . .  axn  n, () x  R.
 x

Ar[ta\i c[: a 1  a 2  . . .  a n  1.
T.L.3733. Determina\i intervalele de concavitate ]i convexitate ]i punctele de infle-
xiune ale func\iilor f definite prin:
a) f (x )  3x 3  1; b) f (x )  2 x ; c) f (x )  lnxx .
x 1

Clasa a XII-a
Probleme selectate de: prof. C[t[lin Spiridon, Craiova

T.L.3734. Se consider[ polinomul f  2X3  (m  1) X2  mX  2m  1  RX.


a) S[ se determine m  R, astfel @nc`t f s[ fie divizibil cu X  1.
b) S[ se determine m  R, astfel @nc`t x 21  x 22  x 23  1  m, unde x 1 , x 2 , x 3 sunt
r[d[cinile polinomului f.
c) Ar[ta\i c[ dac[ m  Z, atunci f nu poate avea toate r[d[cinile @ntregi.
T.L.3735. Se consider[ polinomul f  X4  aX3  3X2  bX  2  RX.
a) S[ se determine a, b  R ]tiind c[ f este divizibil cu (X  1 ) .
2

b) Pentru a  4 ]i b  10, s[ se determine r[d[cinile lui f .


c) S[ se calculeze x 31  x 32  x 33  x 34 @n func\ie de a ]i b, unde x 1 , x 2 , x 3 , x 4 sunt
r[d[cinile lui f.
T.L.3736. Se consider[ polinomul f  X3  mX2  3X  m  Q X.
a) S[ se determine r[d[cinile polinomului f, ]tiind c[ x 1  2  1.
b) S[ se determine m  Q ]tiind c[ x 1 x 2  x 23 , unde x 1 , x 2 , x 3 sunt r[d[cinile
polinomului f .
c) S[ se determine m  Q ]tiind c[ r[d[cinile lui f sunt @n progresie aritmetic[.
T.L.3737. Se consider[ f  2X 3  3X 2  aX  2  Z X.
a) S[ se determine valorile @ntregi ale lui a ]tiind c[ f admite r[d[cini @ntregi.

38 Revista de matematic[ din Craiova


b) Ar[ta\i c[ dac[ a  N, a  3, atunci f nu poate avea toate r[d[cinile reale.
c) Pentru a  3, s[ se determine r[d[cinile lui f.
T.L.3738. Se consider[ polinomul f  X7  X2  X  a  Z7 X.
a) S[ se arate c[ pentru orice b  Z 7 , b  0, are loc b 6  1.
b) S[ se determine num[rul de elemente al mul\imii H  x 2  x  Z 7 .
c) S[ se determine a  Z7 astfel @nc`t f s[ aib[ r[d[cini @n Z 7 .
T.L.3739. Se consider[ polinomul f  2  X 3  a  X  2  Z 3 X.
a) S[ se determine a  Z3 astfel @nc`t f s[ fie divizibil cu X  2.
b) S[ se descompun[ f @n factori ireductibili pentru a  0.
c) S[ se determine valorile lui a pentru care f este reductibil peste Z 3 X.
T.L.3740. Se consider[ polinomul f  X4  2X3  X2  aX  b  RX.
a) S[ se determine a, b  R astfel @nc`t f s[ fie divizibil cu X 2  1.
b) Pentru a  2 ]i b  0 se noteaz[ cu g c`tul @mp[r\irii polinimului f la X 2  1 .
S[ se calculeze: g(1)  g(2)  . . .  g(n), n  N .
c) S[ se determine a ]i b astfel @nc`t ecua\ia f (x )  0 s[ fie reciproc[ ]i s[ se
rezolve @n acest caz.
T.L.3741. Se consider[ polinomul f  R X, cu proprietatea c[:
(x  1)  f (x)  (x  1)  f (x  3)  6x2015  6, () x  R.
a) S[ se calculeze suma coeficien\ilor polinomului.
b) S[ se calculeze f (2 ).
c) S[ se determine restul @mp[r\irii lui f la (X  1 )  (X 2  4 ).
T.L.3742. Se consider[ polinomul f  X3  3X  a  RX.
a) Demonstra\i c[ pentru orice a  R, polinomul f are o singur[ r[d[cin[ real[.
b) S[ se determine a  R ]i r[d[cinile polinomului f, ]tiind c[ acesta are o
r[d[cin[ pur imaginar[.
x2 x2 x2
c) Pentru a  1, s[ se calculeze E  x 2 1x 3  x 3 2x 1  x 1 3x 2 .
T.L.3743. Se consider[ polinomul f  X 4  2X 3  aX 2  bX  c  R X.
a) S[ se calculeze x 21  x 22  x 23  x 24 , unde x 1 , x 2 , x 3 , x 4 sunt r[d[cinile polinomu-
lui f.
b) Pentru a  3 ]i b, c  R, ar[ta\i c[ f nu poate avea toate r[d[cinile reale.
c) Determina\i a, b, c  Q ]tiind c[ restul @mp[r\irii lui f la X  1 este 3 ]i f are
r[d[cina 1  2 . S[ se determine r[d[cinile lui f @n acest caz.
T.L.3744. Se consider[ func\ia f : 0,   R, f (x )  sin 2 x.
a) S[ se determine aria subgraficului func\iei f .
b) S[ se calculeze volumul corpului ob\inut prin rotirea @n jurul axei Ox a grafi-
cului func\iei f.
c) S[ se calculeze lim
n
f 1 f 1 2
n  f n  ...  f n .
n
n

Editura Cardinal 39
T.L.3745. Se consider[ func\ia f : RR, f (x)  x  ln(x2  1).
a) S[ se arate c[ f este bijectiv[.
b) S[ se calculeze aria por\iunii plane delimitate de graficul func\iei f, axa Ox ]i
dreptele de ecua\ii x  0 ]i x  1.
1ln 2
c) S[ se calculeze  f 1 (x )dx.
0
T.L.3746. Se consider[ func\iile f n : (0, )  R, f n (x )  1 , n  N .
x  x2  n2
a) S[ se calculeze aria por\iunii plane delimitate de graficul func\iei f 1 , axa Ox
]i dreptele de ecua\ii x  1 ]i x  2.
b) S[ se calculeze volumul corpului ob\inut prin rotirea @n jurul axei Ox a grafi-
cului func\iei g : 1, 2  R, g(x )  f 2 (x ).
n
n
n 
c) S[ se calculeze lim .
k1 (n  k ) (k  n ) 2  n 4
ln x
T.L.3747. Se consider[ func\ia f : 1,   R, f (x )   e t  3 dt.
0
a) Ar[ta\i c[ f este monoton[.
f (x )
b) Calcula\i lim .
x1 x  1

c) Calcula\i f (e ).
T.L.3748. Se consider[ func\ia f n : 0,  R, f n (x )  ln(xn  1), n  N .
a) S[ se calculeze aria por\iunii plane de limitate de graficul func\iei f 1 , axa Ox
]i dreptele de ecua\ii x  0 ]i x  1.
b) S[ se calculeze volumul corpului ob\inut prin rotirea @n jurul axei Ox a grafi-
cului func\iei g : 0, 1  R, g(x )  f 1 (x ).
1
c) S[ se calculeze lim
n
 f n (x )dx.
0
T.L.3749. Se consider[ func\ia f : (0,  )  R, f (x )  1 .
x(x 2  1 )
a) S[ se calculeze aria por\iunii plane delimitate de graficul func\iei f, axa Ox ]i
dreptele de ecua\ii x  1 ]i x  e.

3

b) S[ se calculeze:  f (sin x )dx.



6
n1
c) S[ se calculeze: lim
n
 f (x )dx.
n

*
* *

40 Revista de matematic[ din Craiova


*********************************************************************
* LUCR{RI DE VERIFICARE *
*********************************************************************

CLASELE V-XII

Clasa a V-a
Lucrarea 1 - (timp de lucru: 50 min.)
Alc[tuit[ de prof. DORINA DR{CEA, Craiova
1. Calcula\i:
a) 7  0, 5  0, (4 )  5  3; b) 14, 4 : 0, 2  0, 4  (0, 04  0, 1 ).
3 12
2. Afla\i x din egalit[\ile: a) 2x  0, 5  3, 75;
b) 4  (x  2, 3 )  1, 25; c) 0, 2 x  0, 04 x  0, 008 x  0, 2 12.
3. Media aritmetic[ a trei numere a, b ]i c este egal[ cu 7, 5. Media aritmetic[ a
primelor dou[ numere este egal[ cu 8, 4 iar primul num[r este de 3 ori mai mare dec`t
al doilea. Afla\i cele trei numere.
4. Calcula\i: a) 43 dm  0, 7 m  1, 2 cm  0, 003 dam;
b) 7 l  0, 2 dm 3  0, 005 m 3; c) 4 ha  2, 5 ari  1420 m 2.
5. Dintr-un dreptunghi cu aria egal[ cu 375 m 2 se @nl[tur[ un dreptunghi cu aria
de 150 m 2 ]i r[m`ne un p[trat care are latura egal[ cu l[\imea dreptunghiului ini\ial.
Afla\i perimetrul dreptunghiului ini\ial.

Lucrarea 2 - (timp de lucru: 50 min.)


Alc[tuit[ de prof. DORINA DR{CEA, Craiova

SUBIECTUL I. Completa\i cu r[spunsurile corecte:


1. 1  1  . . . . . .
2 3
2. 0, (3 )  0, (1 )  9  . . . . . .
3. 30% din 60  . . . . . .
4. 7m 3  . . . . . . l.
5. M a (1, 75; 2; 1, 5 )  . . . . . .
6. Aria p[tratului cu latura egal[ cu 1, 5 m  . . . . . . dm 2 .

Editura Cardinal 41
SUBIECTUL II.
1. Desena\i un cub.
2. Rezolva\i ecua\ia:
1, 5  1 : 0, (6 )  x  0, 1(26 ) : 25  2 : 0, 2  15, 25.
3 198
3. Dup[ ce a cheltuit 25% din suma pe care o avea, Maria mai are 20 de lei. Ce
suma a avut Maria?
4. Diferen\a a dou[ numere este 80, 25, iar unul dintre ele este de cinci ori mai
mare dec`t cel[lalt. Afla\i cele dou[ numere.
5. Afla\i cea mai mic[ ]i cea mai mare frac\ie de forma 1x5 ]tiind c[ 1x53.
y24
6. Ar[ta\i c[ frac\ia n 2
 n este reductibil[, pentru orice n  N.
2n  6

SUBIECTUL III.
1. Un dreptunghi are perimetrul egal cu 32, 8 dam ]i lungimea de trei ori mai
mare dec`t l[\imea.
a) Afla\i lungimea dreptunghiului.
b) Afla\i aria dreptunghiului.
c) Afla\i aria p[tratului care are latura egl[ cu 0, 5 din l[\imea dreptunghiului.
2. Un bazin @n form[ de paralelipiped dreptunghic are L  10 m, l  6 m ]i
h  3 m.
a) Afla\i perimetrul bazei.
b) Afla\i volumul bazinului exprimat @n litri.
c) C`\i litri de ap[ se pun @n bazin, dac[ @n[l\imea apei reprezint[ 70% din
@n[l\imea bazinului.

Clasa a VI-a
Lucrarea 1 - (timp de lucru: 60 min.)
Alc[tuit[ de prof. SIDONIA CAZACU, Craiova
SUBIECTUL I. (30 puncte) - Pe foaia de tez[ se trec doar rezultatele.
5p 1. Rezultatul calculului 2  2  (3 ) este egal cu . . . . . .
5p 2. Dac[ x  6 atunci x  . . . . . .
2 4
5p 3. Dintre numerele a  6 ]i b  4 mai mare este . . . . . .
5p 4. Dac[ 5 kg de struguri cost[ 45 lei atunci 9 kg de struguri cost[ . . . . . . lei.
5p 5. Dac[ @n triunghiul ABC, m(A )  m(B )  70  , atunci m(C )  . . . . . .
5p 6. Construi\i un triunghi ascu\itunghic ABC ]i desena]i @n[l\imea din A.

42 Revista de matematic[ din Craiova


SUBIECTUL II. (30 puncte) - Pe foaia de tez[ se trec rezolv[rile complete.
1. Calcula\i: 3 2  2  (3 ) : (3 ).
2
5p
5p 2. Un obiect cost[ 1200 lei. C`t va costa obiectul dup[ o cre]tere de pre\ cu
15%?
5p 3. Rezolva\i @n R: a) 2x  4  8; b) 2x  3  2.
4. Triunghiul dreptunghic ABC are m(A )  90  , AD  BC, D  BC, DB 
 8 cm ]i m(C )  30 
5p a) Calcula\i m(BAD ).
10p b) Calcula\i DC.

SUBIECTUL III. (30 puncte) - Pe foaia de tez[ se trec rezolv[rile complete.


1. Efectua\i:
5p a) 10  6  10  (12 )  10;
b) 3 28 : (3 26 )  (2 3 ) : 8 6  (5 4 ) : 25 6 .
7 3
5p
5p 2. Rezolva\i @n Z ecua\ia: x  1  x  2  1.
3. #n triunghiul ABC, cu AB  AC , perpendiculara @n B pe dreapta AB inter-
secteaz[ perpendiculara @n C pe AC @n punctul E.
5p a) Ar[ta\i c[ AE este bisectoarea unghiului BAC.
5p b) Dac[ O este mijlocul lui BC afla\i m(AOC ).
5p c) Dac[ BE  BC afla\i m[surile unghiurilor triunghiului ABC.

Lucrarea 2 - (timp de lucru: 50 min.)


Alc[tuit[ de prof. CONSTANTIN CAZACU, Craiova

SUBIECTUL I. (30 puncte) - Pe foaia de tez[ se trec doar rezultatele.


5p 1. Dublul num[rului 8 este egal cu . . . . . .
5p 2. 50% din 20 este egal cu . . . . . .
5p 3. Rezultatul calculului 4  (2 ) este egal cu . . . . . .
5p 4. Dac[ x  2 atunci x  . . ., . . ..
5p 5. M[surile unghiurilor unui triunghi echilateral este egal cu . . . . . . grade.
5p 6. Construi\i mediana BM a unui triunghi ABC dreptunghic @n A.

SUBIECTUL II. (30 puncte) - Pe foaia de tez[ se trec rezolv[rile complete.


5p 1. Calcula\i suma tuturor numerelor @ntregi cuprinse @ntre 30 ]i 25.
5p 2. Un obiect cost[ 780 lei. Cu c`t la sut[ va cre]te pre\ul obiectului pentru ca
obiectul s[ se v`nd[ cu 897 lei?
5p 3. Rezolva\i @n Z ecua\ia: 3(2x  1 )  4(x  1 )  9(x  4 ).

Editura Cardinal 43
5p 4. M[surile unghiurilor unui triunghi sunt propor\ionale cu numerele 2, 7 ]i res-
pectiv 9. Ar[ta\i c[ triunghiul este dreptunghic.
5p 5. Diferen\a dintre un num[r @ntreg ]i modulul s[u este egal[ cu 4. Afla\i nu-
merele @ntregi care @ndeplinesc condi\ia de mai sus.
5p 6. #n triunghiul dreptunghic ABC cu m(A )  90  punctul I este intersec\ia
bisectoarelor interioare ale triunghiului. Afla\i m(BIC ).

SUBIECTUL III. (30 puncte) - Pe foaia de tez[ se trec rezolv[rile complete.


1.
a) Calcula\i: (13  11 )  (3  2 )  (7 ) : (7 )  3(10  2 2 )  (5 )  (3 0 ).
2 3 2
5p
5p b) Afla\i numere @ntregi a ]i b ]tiind c[: 2 a  2  b  3  2.
5p c) Dac[ n  N,afla\i cea mai mare valoare a expresiei
E  (1 ) n  2  (1 ) 2n  3  (1 ) 3n  4  (1) 4n  . . .  n  (1 ) nn .
2. #n triunghiul ABC cu m(A )  90  , m(C )  30  , AB  10 cm, perpendicu-
lara din A pe bisectoarea unghiului B se intersecteaz[ cu BC @n E, iar perpendiculara
din C pe bisectoarea unghiului B se intersecteaz[ cu AB @n F.
5p a) Afla\i perimetrul triunghiului BCF.
10p b) Ar[ta\i c[: FE  AC .

Clasa a VII-a
Lucrarea 1 - (timp de lucru: 2 ore)
Alc[tuit[ de prof. CRISTIANA SEINU, Craiova

SUBIECTUL I. (30 puncte) - Pe foaia de tez[ se trec doar rezultatele.


Completa\i cu r[spunsurile corecte

5p 1. Rezultatul calculului 13a  85a  15a este . . . . . .


2. Calcul`nd (2x  5 ) ob\inem . . . . . .
2
5p
5p 3. Valoarea lui x din ecua\ia 2(x  1 )  42  34 este . . . . . .
5p 4. Diagonala unui p[trat cu lungimea laturii de 15 cm este . . . . . . .
5p 5. #n[l\imea unui triunghi echilateral cu latura de 9 cm este . . . . . .
5p 6. Aria unui triunghi ABC, cu AB  18 cm, AC  10 cm ]i m(BAC )  60  este .
.....

44 Revista de matematic[ din Craiova


SUBIECTUL II. (30 puncte) - Pe foaia de tez[ se trec rezolv[rile complete.

5p 1. Desena\i pe foaia de tez[ un paralelogram ABCD cu AC  BC.


2. Calcula\i: (3a  2 )(3a  2 )  2(a  5)  5  (2  a ).
2
5p

5p 3. Rezolva\i @n R ecua\ia: 2x  3  3x  2  4x  3  5 .
3 2 4 12
5p 4. Rezolva\i inecua\ia @n Z: 6(x  3 )  10  2(2x  1 ).
5p 5. Un romb are diagonalele de 32 cm ]i respectiv 24 cm. Afla\i perimetrul ]i
aria rombului.
5p 6. Fie triunghiul dreptunghic ABC, cu m(A )  90 . }tiind c[ AD  BC,
D  (BC ), AD  12 cm ]i DC  16 cm, afla\i laturile triunghiului ]i aria.

SUBIECTUL III. (30 puncte) - Pe foaia de tez[ se trec rezolv[rile complete.

1. P[tratul ABCD din figur[ are latura AB  10 cm. Punctul M  (AB ) astfel
@nc`t AM  6 cm. Determina\i:
D C
5p a) lungimea diagonalei BD;
5p b) aria triunghiului BMD;
5p c) distan\a de la punctul M la diagonala BD.

A M B

2. Trapezul isoscel ABCD, cu AB  CD are m A  m B  60  ]i diagonala


AC  BC. }tiind c[ BC  12 cm, afla\i:
5p a) diagonalele ]i perimetrul trapezului;
5p b) natura triunghiului MDC, unde AD  BC  M;
5p c) raportul dintre aria triunghiului ABC ]i aria trapezului ABCD.

Lucrarea 2 - (timp de lucru: 2 ore)


Alc[tuit[ de prof. CRISTIANA SEINU, Craiova

SUBIECTUL I. (30 puncte) - Pe foaia de tez[ se trec rezolv[rile complete.


5p 1. Rezultatul calculului (12  34 : 2 )  3  15 este egal cu . . . . . .
5p 2. Calcul`nd sin 30   cos 60  ob\inem . . . . . .
5p 3. Forma descompus[ a expresiei algebrice 4x 2  1 este . . . . . .
5p 4. #ntr-o urn[ sunt 16 bile albe ]i 4 bile negre. Probabilitatea ca lu`nd la
@nt`mplare o bil[ aceasta s[ fie alb[, este egal[ cu . . . . . .
5p 5. Aria unui p[trat cu latura de 25 cm este egal[ cu . . . . . . cm 2 .

Editura Cardinal 45
5p 6. Media geometric[ a numerelor a  10  1 ]i b  10  1 este . . . . . .

SUBIECTUL II. (30 puncte) - Pe foaia de tez[ se trec rezolv[rile complete

5p 1. Desena\i pe foaia de tez[ rombul MNPR cu MP  RN.


10p 2. Demonstra\i c[ num[rul
a  ( 6  2 ) 2  ( 6  3 )  ( 6  3 )  (2 3  1 ) 2 este un num[r ra\ional.
5p 3. Rezolva\i @n mul\imea N inecua\ia: 0, 2x  2  0, 3x  2, 3.
10p 4. Un triunghi dreptunghic are catetele egale cu x  1 ]i x  2. Afla\i valoarea
num[rului real x dac[ ipotenuza este egal[ cu x  3.

SUBIECTUL III. (30 puncte) - Pe foaia de tez[ se trec rezolv[rile complete.

1. Un elev prime]te de la p[rin\i o sum[ de bani de cheltuial[ pentru o excursie


de trei zile. El cheltuie banii astfel: @n prima zi cheltuie 30% din banii primi\i; a doua
zi cheltuie 5 din banii primi\i; a treia zi cheltuie ultimii 93 de lei.
16
5p a) Afla\i suma de bani primit[ de elev.
5p b) Dac[ elevul a primit 240 de lei, afla\i c`t a cheltuit @n primele dou[ zile
de excursie.
5p c) Un alt elev a primit aceea]i suma de bani pentru excursie. El @]i propune
s[ cheltuie banii @n cele trei zile @n p[r\i direct propor\ionale cu numerele naturale
consecutive, astfel @nc`t p[r\ile s[ fie numere @ntregi pozitive. C`te variante sunt
posibile ?
Este adev[rat c[ oricare variant[ ar fi, elevul cheltuie @n a doua zi 80 de lei?

2. #n figura al[turat[ ave\i triunghiul ABC, cu m(BAC )  90  ]i tg(C )  3 .


4
5p a) Dac[ BC  30 cm, afla\i perimetrul triunghiului ABC.
5p b) Punctul M este mijlocul lui BC.
Perpendiculara @n M pe BC intersecteaz[ dreapta C
AC @n punctul E. Ar[ta\i c[ CE  18, 75 cm.
5p c) Calcula\i aria patrulaterului ABME.
M
E
A B

46 Revista de matematic[ din Craiova


Clasa a VIII-a
Lucrarea 1 - (timp de lucru: 120 min.)
Alc[tuit[ de prof. CONSTANTIN BASARAB, Craiova

SUBIECTUL I. (30 puncte) - Pe foaia de tez[ se trec numai rezultatele.


2
5p 1. Rezultatul calculului  1  2 2 este . . . . . .
2
5p 2. Dup[ o reducere cu 20% pre\ul unui obiect este 32 lei. Pre\ul ini\ial era de . .
. . . . lei.
3. Scris[ cu ajutorul intervalelor mul\imea A  x  R  (x  1 )  9 este egal[
2
5p
cu . . . . . .
5p 4. Suma dintre raza cercului @nscris ]i raza cercului circumscris unui triunghi
echilateral este 4 3 cm. Aria triunghiului este . . . . . . cm 2 .
5p 5. Dac[ aria lateral[ a unui cilindru circular drept este 12 cm 2 , iar aria total[
20 cm 2 , volumul lui este . . . . . . cm 3
5p 6. #n tabelul al[turat sunt reprezentate
Nota 4 5 6 7 8 9 10
rezultatele ob\inute de elevii unei clase la o
testare. Media clasei este . . . . . . Nr. 2 4 3 5 6 3 2
elevi

SUBIECTUL II. (30 puncte) - Pe foaia de tez[ se trec rezolv[rile complete.

5p 1. Desena\i un trunchi de piramid[ patrulater[ regulat[ ABCD A  B  C  D  .


5p 2. Ar[ta\i c[ 5 n3  2 n  125 este divizibil cu 45, oricare ar fi n  N.
5p 3. Dup[ ce a parcurs 2 dintr-un drum, un c[l[tor a constatat c[ mai are de
5
parcurs 14 km p`n[ la jum[tatea drumului. Afla\i lungimea drumului.
2 3
5p 4. Calcula\i: 1   7  6  7.
3 1 2 7
5. Se consider[:
E(x)  (x  2 ) 2  (x  1 )(2x  3 )  2x2  9x  5, unde x  R.
5p a) Ar[ta\i c[: E(x)  x2  2x  2, () x  R.
5p b) Determina\i valoarea minim[ a lui E(x ) c`nd x  R.

Editura Cardinal 47
SUBIECTUL III. (30 puncte) - Pe foaia de tez[ se trec rezolv[rile complete.

1. #n figura al[turat[ este reprezentat triunghiul C


dreptunghic ABC ]i trei semicercuri av`nd drept diametre
laturile triunghiului. }tiind c[ AB  16 cm ]i AC  12 cm se
cere:
B
5p a) Lungimea lui BC. A
5p b) Aria suprafe\ei care se afl[ @n interiorul
semicercului de diametru BC ]i @n exteriorul triunghiului
ABC.
5p c) Compara\i aria supafe\ei ha]urate cu aria triunghiului ABC.

2 VABCD este o piramid[ patrulater[ regulat[ cu @n[l\imea VO  6 2 cm ]i


unghiul dintre muchia lateral[ ]i planul bazei 45  .
5p a) Ar[ta\i c[ AB  12 cm.
5p b) Calcula\i aria lateral[ ]i volumul piramidei.
5p c) Afla\i aria triunghiului determinat de centrele de greutate a dou[ fe\e later-
ale opuse ]i centrul bazei.

Lucrarea 2- (timp de lucru: 120 min)


Alc[tuit[ de prof. MARLENA BASARAB, Craiova

SUBIECTUL I. (30 puncte) - Pe foaia de tez[ se trec numai rezultatele.

1. Rezultatul calculului (2  5 )  2 este . . . . . .


2
5p
5p 2. #ntr-o urn[ sunt 7 bile albe ]i 8 bile negre ]i se extrage o bil[. Probabilitatea
ca bila extras[ s[ fie alb[ este . . . . . .
5p 3. Cel mai mare num[r natural de forma 4x3 divizibil cu 3 este . . . . . .
5p 4. O sfer[ cu diametrul 6 cm are aria . . . . . . cm 2 .
5p 5. Aria unui p[trat cu diagonala 4 2 cm este . . . . . . cm 2 .
5p 6. Dac[ 4 muncitori termin[ o lucrare @n 12 zile, atunci 6 muncitori termin[
lucrarea @n . . . . . . zile.

SUBIECTUL II. (30 puncte) - Pe foaia de tez[ se trec rezolv[rile complete.

5p 1. Desena\i un trunchi de con circular drept cu sec\iunea axial[ ABB  A  .


2. Un automobil a parcurs o distan\[ @n trei zile astfel: @n prima zi a parcurs
35% din drum, a doua zi 20% din distan\a r[mas[, iar a treia zi restul de 624 km.

48 Revista de matematic[ din Craiova


5p a) C`\i km are @ntrega distan\[?
5p b) C`\i km a parcurs @n a doua zi?
3. Fie func\ia f : RR, f (x)  (a  2)x  3a  1, a  R.
5p a) Pentru ce valori ale lui a, graficul lui f trece prin A(a; 1 )?
5p b) Pentru a  3 reprezenta\i grafic f ]i afla\i distan\a de la originea sistemului
la grafic.
5p c) Pentru a  4, calcula\i suma S  f (10 )  f (9 )  . . .  f (10 ).

SUBIECTUL III. (30 puncte) - Pe foaia de tez[ se trec rezolv[rile complete.

1. #n figura al[turat[ este schi\a unei gr[dini


dreptunghiulare ABCD, unde AB  24m, BC  18 m, P D N 3 C
este mijlocul lui AD, M este mijlocul lui AB, iar
NC  3 m. Zona ha]urat[ este cu gazon, iar cealalt[ cu
flori. P 18
5p a) Compara\i aria suprafe\ei cultivate cu flori cu 9
aria suprafe\ei cu gazon.
A 12 M 12 B
5p b) Calcula\i perimetrul suprafe\ei cu gazon.
5p c) Determina\i distan\a de la N la MP.

2. Fie un con circular drept cu raza 9 cm ]i @n[l\imea 12 cm.


5p a) Afla\i aria lateral[ ]i volumul conului.
5p b) La ce distan\[ de baza conului trebuie f[cut[ o sec\iune paralel[ cu
aceasta, astfel @nc`t aria lateral[ a conului mic ob\inut s[ fie o treime din aria lateral[ a
trunchiului de con?
5p c) Dac[ M ]i N sunt pe cercul bazei ]i m MN  120  , afla\i distan\a de la
v`rful conului la MN.

Clasa a IX-a
Lucrarea 1 - (timp de lucru: 90 min.)
Alc[tuit[ de prof. C{T{LIN CRISTEA, Craiova

1. Se consider[ func\ia f : R  R, f (x )  2x  6. Calcula\i:


a) f (1 )  f (2 )  . . .  f (6 ); b) f ( 1 )  f ( 2 )  . . .  f ( 10 ).

2. Afla\i m  R ]tiind c[ punctul A 2015 m , 1 apar\ine graficului func\iei


f : R  R, f x  x  2014.
( )
3. Determina\i func\ia de gradul al doilea al c[rei grafic trece prin punctele
A(1, 2 ), B(3, 0 ), C(1, 12 ).

Editura Cardinal 49
4. S[ se demonstreze c[ dreapta de ecua\ie y  2x  3 este tangent[ la parabola de
ecua\ie y  x 2  4x  12.
5. Not[m cu x 1 ]i x 2 solu\iile ecua\iei x 2  x  1  0.
a) Verifica\i c[: x 21  x 22  x 1  x 2  2.
b) Ar[ta\i c[ num[rul N  x 51  x 52 este natural.
     
6. Fie vectorii u  3 ai  3 j ]i v  3 i  (a  2 ) j , unde a este un num[r real.
 
a) Afla\i a ]tiind c[ vectorii u ]i v sunt opu]i.
 
b) Afla\i a ]tiind c[ vectorii u ]i v sunt coliniari.
7. S[ se arate c[: 4 sin x cos x cos 2x  sin 4x, () x  R.
8. Calcula\i:
a) sin 2 25   sin 2 65  .
b)
(sin 1   cos 1  ) 2  (sin 5   cos 5  ) 2  (sin 85   cos 85  ) 2  (sin 89   cos 89  ) 2 .

Lucrarea 2 - (timp de lucru: 90 min.)


Alc[tuit[ de prof. C{T{LIN CRISTEA, Craiova

1. Se consider[ func\ia f : R R, f (x)  2x  1.


a) Reprezenta\i grafic func\ia f.
b) Calcula\i (f  f  f )(10 ).
f (x )
c) Rezolva\i @n R inecua\ia:  0.
f (x )  3
2. Se dau func\iile f, g : R  R definite prin
 2x, dac[ x  2
f (x )  x  1, g(x )   .
 x  1, dac[ x  2
a) Calcula\i (f  f )(2015 ) ]i (g  g )(1 ).
b) Calcula\i g  f ]i g  g.
3. Fie func\ia f : R  R, f (x )  (a  2 )x 2  (a  1 )x  a, unde a este un num[r real.
a) Pentru a  3, rezolva\i @n R ecua\ia f (x )  0.
b) Pentru a  1, afla\i valoarea maxim[ a func\iei f.
c) Pentru a  0, rezolva\i @n R inecua\ia f (x)  1.

4. Ar[ta\i c[ tg   x  cos x  sin x , ( ) x  R cu sin x  cos x  0.


4 cos x  sin x
5. Dac[ a  sin 2 1   sin 2 2   . . .  sin 2 88   sin 2 89  , ar[ta\i c[ num[rul 2  a este
natural.

50 Revista de matematic[ din Craiova


Clasa a X-a
Lucrarea 1 - (timp de lucru: 90 min.)
Alc[tuit[ de prof. DANIEL CIURCEA, Craiova

SUBIECTUL I. (30 puncte)


5p 1. Calcula\i 10% din 37.
5p 2. Calcula\i: 5!  4!  3!.
5p 3. Dac[ 17% din pre\ul unui obiect este egal cu 204 lei, afla\i pre\ul obiectului.
5p 4. Afla\i probabilitatea de a ob\ine un num[r prim la aruncarea unui zar.
5p 5. Afla\i num[rul func\iilor pare f : 3, 2, 1, 0, 1, 2, 3  C 17 , C 27 , C 37 .
5p 6. Determina\i coordonatele centrului de greutate al triunghiului ABC, unde:
A(1, 2 ), B(1, 5 ), C(3, 7 ).

SUBIECTUL II. (30 puncte)


n
1. Fie dezvoltarea B(x )  x 2  1x , x  0, n  N  , n  3.
5p a) Afla\i valorile lui n  N , n  3, pentru care suma primilor trei coefi-
cien\i ai dezvolt[rii este egal[ cu (n  4 ) .
2

5p b) Pentru n  15 afla\i rangul termenului care nu @l con\ine pe x.


5p c) Pentru n  17 afla\i rangul termenului care @l con\ine pe x 7 .
2. Calcula\i sumele:
5p a) S  C 12015  2  C 22015  3  C 32015  . . .  2015  C 2015
2015.

5p b) T  1  2  3  . . .  2014 .
2! 3! 4! 2015!
0
C C 1
C 2015
5p c) U  2015  2015  . . .  2015 .
1 2 2016

SUBIECTUL III. (30 puncte)


1. Fie punctele A(3, 0 ), B(0, 7 ) ]i C(4, 2 ).
5p a) Afla\i perimetrul triunghiului ABC.
5p b) Afla\i aria triunghiului ABC.
5p c) Afla\i distan\a de la punctul A la dreapta BC.
2. Fie punctele A(1, m), B(0, m  3) ]i C(2, 1), m  R.
5p a) Afla\i pantele dreptelor AB ]i BC.
5p b)Determina\i valorile lui m  R pentru care punctele A, B, C sunt coliniare.
5p c)Scrie\i ecua\ia dreptei ce trece prin origine ]i este paralel[ cu dreapta AB.

Editura Cardinal 51
Lucrarea 2 - (timp de lucru: 90 min.)
Alc[tuit[ de prof. DANIEL CIURCEA, Craiova

SUBIECTUL I. (30 puncte)


5p 1. Calcula\i probabilitatea de a ob\ine o bil[ alb[ dac[ extragem o bil[ la @nt`m-
plare dintr-o urn[ ce con\ine 16 bile albe ]i 7 bile negre.
5p 2. Afla\i num[rul ce reprezint[ 25% din 1476.
5p 3. Determina\i c`te func\ii injective f : 1, 2, 3  5, 7, 9, 11 exist[.

 

5p 4. Determina\i coordonatele punctului M cu proprietatea c[ AM  2MB ]i
A(1, 3 ), B(7, 9 ).
A3
5p 5. Calcula\i valoarea frac\iei 72 .
A4
5p 6. Afla\i panta dreptei AB, unde A(1, 0 ) ]i B(2, 3 ).

SUBIECTUL II. (30 puncte)

5p 1. a) Demonstra\i c[: 2015!  N.


1946!  69!
b) Afla\i num[rul termenilor ra\ionali din dezvoltarea ( 4 2  3 4 ) .
100
5p
100
5p c) Afla\i rangul celui mai mare termen al dezvolt[rii 1  4 .
5 5
2. Trei sportivi trag c`te un foc asupra unei \inte, ]ansele de a o nimeri fiind
respectiv 2 , 2 ]i 3 . Afla\i probabilitatea evenimentului ca \inta s[ fie lovit[:
3 5 4
5p a) cel pu\in o dat[;
5p b) exact o dat[;
5p c) o dat[ sau de dou[ ori.

SUBIECTUL III. (30 puncte)


1. Consider[m punctele A(3, 1 ), B(1, 5 ) ]i C(3, 0 ).
5p a) Scrie\i ecua\ia medianei din A a triunghiului ABC.
5p b) Scrie\i ecua\ia dreptei BC.
5p c) Scrie\i ecua\ia @n[l\imii din A a triunghiului ABC.
2. Consider[m punctele A(2, 3 ), B(1, 5 ), C(4, 7 ) ]i D(3, 1 ).

  
5p a) Calcula\i coordonatele vectorilor AB ]i BC.
5p b) Ar[ta\i c[ ABCD este paralelogram.

  
5p c) Dac[ punctul M are proprietatea c[ AM  3MB, afla\i coordonatele cen-
trului de greutate al triunghiului MCD.

52 Revista de matematic[ din Craiova


Clasa a XI-a
Lucrarea 1 - (timp de lucru: 90 min.)
Alc[tuit[ de prof. FELICIAN PREDA, Craiova

SUBIECTUL I.
 a b 
1. Fie mul\imea M    a, b, c, d  Z .C`te matrice din mul\imea M
 c d 
au suma elementelor egal[ cu 1.
 mx  y  z  0

2. Fie m  R ]i sistemul:  x  3y  2z  0 . S[ se determine m astfel @nc`t siste-
 x  y  4z  0

mul s[ aib[ ce pu\in dou[ solu\ii.
3. Calcula\i lim sin x cos 1x .
x0
x2  1
4. Deriva\i func\ia f : R1  R : f (x )  .
x1
5. Folosind ]irul lui Rolle, separa\i r[d[cinile reale ale ecua\iei:
12x 5  15x 4  20x 3  30x 2  17  0.
6. Discuta\i @n func\ie de a  R, num[rul r[d[cinilor reale ale ecua\iei:
ln x 2  ax  0.

SUBIECTUL II.
 x ny 
1. Fie n  Z. Definim mul\imea A n    x, y  Z .
 y x 
a) Ar[ta\i c[ elementele lui A n comut[ @n raport cu @nmul\irea matricelor.
b) Exist[ X, Y  A 1 nenule cu XY  O 2 ?
c) Dac[ X, Y  A 2 ]i XY  O 2 , atunci X  O 2 sau Y  O 2.
2. S[ se studieze continuitatea func\iilor g  f ]i f  g unde f : RR, g : RR
sunt definite prin:
 1, x  0

f (x )   0, x  0 ]i g(x )  x 2  4x  3.
 1, x  0


Editura Cardinal 53
SUBIECTUL III.
 ax  y  2z  0

1. Se consider[ sistemul:  x  ay  z  0 .
 2x  2y  az  0

a) Pentru ce valori ale lui a sistemul are solu\ie unic[?
b) S[ se rezolve sistemul pentru a  2.
2. S[ se g[seasc[ punctele de extrem ale func\iei f : 1;   R definit[ prin:
f (x )  e  4  x 3  1 .
3x

Lucrarea 2 - (timp de lucru: 90 min.)


Alc[tuit[ de prof. FELICIAN PREDA, Craiova

SUBIECTUL I.
 2x  1, x  0
1. Ar[ta\i c[ func\ia f (x )   are proprietatea lui Darboux.
 x  1 , x  0
2. Ar[ta\i c[ ecua\ia x  3 x  1 are cel pu\in o solu\ie @n intervalul (1, 0 ).
3. Calcula\i derivata func\iei f : (0,  )  R, f (x )  x  ln x .
x  ln x
4. Afla\i punctele unghiulare ]i punctele de @ntoarcere ale func\iei
 x 2  1, x  0
f : R  R, f (x )   .
 cos x, x  0
5. Fie f : R  R, f (x )  e x sin x  e x cos x. Verifica\i dac[:
f  (x )  2 f  (x )  2 f (x )  0, () x  R.
6. Afla\i a  R , astfel @nc`t: ax  x  1, () x  R.

SUBIECTUL II.

1. S[ se discute dup[ parametrii reali a, b, c ]i s[ se rezolve sistemul:


 xyz1

 ax  by  (a  b )z  c .
 a 2 x  b 2 y  (a 2  b 2 )z  c 2

2. S[ se reprezinte grafic func\ia f : R  R, f (x )  (x  1 ) arcsin x2 .
x21

54 Revista de matematic[ din Craiova


SUBIECTUL III.
1. }tiind c[ r[d[cinile x 1 , x 2 , x 3 ale ecua\iei x3  x 2  ax  b  0, a, b  R sunt
x1 x2 x3
reale, s[ se arate c[ acestea sunt egale dac[ ]i numai dac[: x 3 x 1 x 2  0.
x2 x3 x1
S[ se rezolve @n acest caz ecua\ia dat[ ]i s[ se determine a ]i b.
2. S[ se reprezinte grafic func\ia f : R  R, definit[ prin:
 2
 x , x  1
f (x )   x  1 .

 x, x  1

Clasa a XII-a
Lucrarea 1 - (timp de lucru: 90 min.)
Alc[tuit[ de prof. CRISTINA SPIRIDON, Craiova

Subiectul I (30 puncte)


5p 1. S[ se determine num[rul complex z  C astfel @nc`t: z 2  z 2  8  2i.
5p 2. S[ se determine a  R ]tiind c[: x2  (a  2) x  a  1  0, () x  R.
5p 3. S[ se rezolve @n mul\imea numerelor reale ecua\ia: 2  3 x  3 1x  7.
12
5p 4. Se consider[ dezvoltarea 3 x  1x unde x  R  . S[ se determine termenul
care nu con\ine x.
5p 5. #n reperul cartezian xOy se consider[ punctele A(1, 3 ), B(2, 1 ), ]i C(3, 1 ).
S[ se scrie ecua\ia @n[l\imii duse din v`rful A @n triunghiul ABC.
5p 6. Fie   0,  astfel @nc`t cos   3 . S[ se calculeze sin 3.
2 5

Subiectul II (30 puncte)

 4x  y  2z  0

1. Se consider[ sistemul  mx  2y  z  0 , m  R.
 11x  4y  mz  0

5p a) S[ se calculeze det A, unde A este matricea sistemului.
5p b) S[ se determine m  R ]tiind c[ sistemul admite solu\ii nenule.
5p c) Pentru m  1, s[ se rezolve sistemul.

Editura Cardinal 55
2. Pe mul\imea numerelor reale se consider[ legea de compozi\ie
x  y  2xy  2x  ay  b, a, b  R.
5p a) Determina\i a, b  R ]tiind c[ legea dat[ s[ fie asociativ[ ]i comutativ[.
5p b) Pentru a  2 ]i b  1, afla\i simetricul lui 0 @n raport cu legea dat[.
5p c) Pentru a  2 ]i b  1, s[ se calculeze x  x  . . .  x.
2015 ori

Subiectul III (30 puncte)

1. Se consider[ func\ia f : R2  R, f (x )  x  2x  1 .


2

x2
5p a) S[ se determine asimptotele la graficul func\iei f.
x
f (x )
5p b) S[ se calculeze lim
x x .
5p c) S[ se arate c[: f (x )  f 1x  4, ( ) x  (2,  ).
2. Se consider[ func\ia f : 0, 1  R, f (x )  x 2  4  x 2 .
5p a) S[ se calculeze volumul corpului ob\inut prin rotirea @n jurul axei Ox, a
graficului func\iei f.
1
5p b) S[ se calculeze:  f (x )dx.
0 x
 f (t )dt
0
5p c) S[ se calculeze: lim .
x0 x3

Lucrarea 2 - (timp de lucru: 90 min.)


Alc[tuit[ de prof. C{T{LIN SPIRIDON, Craiova

Subiectul I (30 puncte)

5p 1. S[ se calculeze: log 2 (4  6 )  log 2 (4  6 )  log 2 5.


5p 2. S[ se determine m  R ]tiind c[ graficul func\iei
f : RR, f (x)  x2  (m  3) x  m  2 este tangent axei Ox.
5p 3. S[ se determine solu\iile reale ale ecua\iei: x 2  x  2  x 2  x.
5p 4. S[ se determine num[rul func\iilor f : 1, 2, 3, 4, 5  1, 2, 3, 4, 5 cu propri-
etatea c[ f (1 )  f (2 )  5.
5p 5. S[ se determine a  R ]tiind c[ vectorii
     
v (2a  1 ) i (a  2 ) j ]i u  (a  1 ) i (a  8 ) j sunt coliniari.
5p 6. S[ se calculeze raza cercului @nscris @ntr-un triunghi care are lungimile laturi-
lor 5, 7 ]i 8.

56 Revista de matematic[ din Craiova


Subiectul II (30 puncte)

m1 2 2
1. Se consider[ matricea A  2 m1 2 , m  R.
2 2 m1

a) S[ se arate c[: det A  (m  5 ) (m  1 ) .


2
5p
5p b) Pentru m  0, s[ se calculeze A 1 .
5p c) Pentru m  1, s[ se calculeze An , n  N .
2. Se consider[ polinomul f  X4  3X3  2X2  aX  b  RX.
5p a) S[ se determine a, b  R ]tiind c[ f admite r[d[cina x 1  i.
5p b) Pentru a  3 ]i b  1, s[ se determine r[d[cinile polinomului.
5p c) S[ se calculeze x 21  x 22  x 23  x 24 , unde x 1 , x 2 , x 3 , x 4 sunt r[d[cinile lui f.

Subiectul III (30 puncte)

1. Se consider[ func\ia f : RR, f (x)  x  ln(x2  1).


5p a) S[ se arate c[ f este bijectiv[.
5p b) S[ se determine punctele de inflexiune ale func\iei f.
f 1 (x )
5p c) S[ se calculeze lim
x x .
1
2. Fie I n  xn dx.
0 x2  4x  5
5p a) S[ se calculeze I 0 ]i I 1 .
5p b) S[ se arate c[: I n2  4I n1  5I n 1 , ( ) n  N .
n1
5p c) S[ se calculeze lim I , lim nI n ]i lim
n n n
n 1  nI n .
n 10

*
* *

Editura Cardinal 57
************************************************************ *********
* PROBLEME PROPUSE *
*********************************************************************

GIMNAZIU

Clasa a V-a
G.1073. a) Da\i exemplu de un num[r natural nenul n care s[ fie p[trat perfect cu
propriet[\ile:
(1) 3n este cub perfect;
(2) 4n este puterea a patra a unui num[r natural;
(3) 5n este puterea a cincea a unui num[r natural.
b) Ar[ta\i c[ exist[ o infinitate de numere n cu propriet[\ile de la a).
Prof. CARMEN TERHECI, Craiova
Prof. CAMELIA DAN{, Craiova

G.1074. Dac[ a  b  ba , ar[ta\i c[: ab  (a  b ) .


2
2 4
Prof. GRIGORE HORTOPAN, Rugi, Gorj
Prof. DORU HORTOPAN, Rugi, Gorj
G.1075. Determina\i numerele naturale ]i nenule n, ]tiind c[ ultima cifr[ a num[-
rului N  n!  87 este 7, unde n!  1  2  . . .  n.
Prof. DOINA STOICA , Arad
Prof. MIRCEA MARIO STOICA, Arad
G.1076. Dac[ S  1  5  5 2  5 3  . . .  5 2014 , ar[ta\i c[ 4S  1se poate scrie ca
sum[ de cinci numere naturale consecutive.
Prof. MIRCEA POPESCU, Tg. C[rbune]ti, Gorj

G.1077. C`te numere de forma abc se divid cu 7, dac[ a  b  c se divide cu 7?


Prof. GRIGORE HORTOPAN, Rugi, Gorj
Prof. DORU HORTOPAN, Rugi, Gorj

_______________________________________
Se primesc solu\ii p`n[ la 15.07.2015 (data po]tei)

58 Revista de matematic[ din Craiova


Clasa a VI-a
G.1078. Stabili\i c`te tipuri de triunghiuri necongruente cu lungimile laturilor nu-
mere @ntregi au perimetrul 13?
Prof. MARIANA LILIANA POPESCU, Suceava
G.1079. a) Dac[ n  12345654321, ar[ta\i c[ frac\ia n n1 n este reduc-
2  5  124
tibil[.
b) S[ se determine num[rul natural n astfel @nc`t 5x  4  N.
(2x  1 )(3x  1 )
Prof. MARIANA COAND{, Gala\i
G.1080. Ar[ta\i c[ frac\ia F  200...03 este ireductibil[, ]tiind c[ cifra 0 apare de
500...07
2013 ori.
Prof. IONEL TUDOR, C[lug[reni
Prof. VIORICA DOGARU, Giurgiu

G.1081. a) Ar[ta\i c[ frac\ia ordinar[ 3  n  2 este ireductibil[ pentru orice n  N.


4n3
b) Demonstra\i c[ mul\imea

A  a  N  a  3n  2  13m  10 , m, n  N este finit[.


4n  3 4m  3
Prof. VERONICA GRIGORE, Gala\i
G.1082. Fie triunghiul ABC cu AB  AC ]i E  (AB ), F  (AC ) astfel @nc`t BE 
 AF. Dac[ M este mijlocul lui (EF ) ]i AM  BC  N, ar[ta\i c[ M este mijlocul lui
(AN ).
Prof. MIRCEA POPESCU, Tg. C[rbune]ti, Gorj

Clasa a VII-a
G.1083. Determina\i numerele de forma ab  1  2  . . .  (a  b ).
Prof. GRIGORE HORTOPAN , Rugi, Gorj
Prof. DORU HORTOPAN, Rugi, Gorj
G.1084. Fie ABC un triunghi ]i AM mediana corespunz[toare v`rfului A, cu M 
 (BC ). Demonstra\i c[ simediana din A a triunghiului ABC, simediana din B a triun-
ghiului AMB ]i simediana din C a triunghiului AMC sunt concurente. (simediana unui
triunghi este simetrica medianei fa\[ de bisectoarea interioar[ dus[ prin acela\i v`rf al
triunghiului).
Prof. ION P{TRA}CU, Craiova

Editura Cardinal 59
G.1085. a) Ar[ta\i c[: 1  1  1 .
2 3 2 5 5 3 3 5
b) Determina\i num[rul @ntreg n cu proprietatea n  a  n  1, unde:
a  2  (5  3  3  5 ) 1 .
c) Determina\i prima cifr[ dup[ virgul[ a num[rului
a  2  (5  3  3  5 ) 1 .
Prof. ROMEO ZAMFIR, Gala\i

G.1086. Dac[ a 1 , a 2 , . . ., a 1002 sunt numere naturale impare, demonstra\i c[:

a 1002
1  a 1002
2  . . .  a 1002
1002  R  Q.
Prof. CERASELA BOCIU, Timi]oara

G.1087. #n triunghiul ABC construim bisectoarea CD, mediana (CE ) D, E 


 AB ), I centrul cercului @nscris @n triunghi ]i G centrul de greutate al triunghiului.
(
2
Dac[ CI  1  BD  2 , ar[ta\i c[ punctele D, E, I, G se afl[ pe acela]i cerc.
CE BC 3
Prof. GHEORGHE F. MOLEA, Curtea de Arge]

Clasa a VIII-a
G.1088. Determina\i numerele reale x, astfel @nc`t raportul 3x2  x  2 s[ fie num[r
2

x x2
natural.
Prof. NECULAI STANCIU, Buz[u

G.1089. Ar[ta\i c[ num[rul A  2 33  99  2 22  3267  2 11 se poate scrie ca sum[ a


dou[ cuburi perfecte.
Prof. IONEL TUDOR, C[lug[reni, Giurgiu
Prof. VIORICA DOGARU, Giurgiu
G.1090. Fie a, b, c  N  astfel @nc`t 3ab  2c 2 . Ar[ta\i c[ num[rul a 3  b 3  c 3 este
compus.
Prof. DAN LUCIAN GRIGORIE, Craiova
G.1091. #n prisma patrulater[ regulat[ ABCDA  B  C  D  fie P simetricul lui A fa\[
de B ]i Q piciorul bisectoarei unghiului C al triunghiului D  CB. Demonstra\i c[
dreapta PQ trece prin centrul fe\ei ADD  A  dac[ ]i numai dac[ AA  AB  3 .
Prof. PETRU BRAICA, Satu Mare

60 Revista de matematic[ din Craiova


G.1092. #n centrul O al triunghiului echilateral ABC cu lungimea laturii a, se ridic[
perpendiculara OD pe planul lui. Dac[ M ]i N sunt mijloacele segmentelor (AB )
respectiv, iar unghiul dintre planele (DMN ) ]i (ABC ) are m[sura 30  , calcula\i:
a) d(O, AD ); b) sin BED , CED , unde E  MN  AO.
Prof. GHEORGHE F. MOLEA, Curtea de Arge]

LICEU

Clasa a IX-a
L.1093. Dac[ a, b, c  (0,  ) ]i a  b  c  S, demonstra\i c[:
a  b  c  9S .
bc  1 ca  1 ab  1 S 2  9
Prof. dr. DORIN M{RGHIDANU, Corabia, Olt
L.1094. Rezolva\i @n mul\imea numerelor reale ]i pozitive sistemul de ecua\ii:

 xyz5
 1 1 .
  y  1  1, 5
 x z1
Prof. GHEORGHE F. MOLEA, Curtea de Arge]
L.1095. Fie ]irul de numere strict pozitive (x n ) n N , cu proprietatea:
x n (x n1  x n1 )  2x n1  x n1 , ( ) n  1.
Ar[ta\i c[: x 0  x 1 .
Prof. DAN SECL{MAN, Craiova
L.1096. a) Ar[ta\i c[ num[rul A  5  32  cos 2 9   32  cos 4 9  este natural.
b) Rezolva\i @n mul\imea numerelor reale ecua\ia:

x x  48  cos 9   8  cos 27   4  sin 37  cos



73  .
sin 10 
Prof. IONEL TUDOR, C[lug[reni, Giurgiu
L.1097. #n triunghiul ABC not[m cu I a centrul cercului ex@nscris tangent laturii
(BC ) @n A 2 , dreptei AC @n B 2 ]i dreptei AB @n C 2 . Ar[ta\i c[ cercurile circumscrise tri-
unghiurilor AI a A 2 , BI a B 2 , ]i CI a C 2 mai au un punct comun diferit de I a .
Prof. PETRU BRAICA, Satu Mare

Editura Cardinal 61
Clasa a X-a
L.1098. Demonstra\i c[ dac[ a  0, a  1 este fixat, atunci exist[ o infinitate de pe-
rechi (x, y )  R  R @n fiecare din cazurile:
a) log a (x  y )  log a x  log a y; b) log a (x  y )  log a x  log a y .
Prof. DANA HEUBERGER, Baia Mare

L.1099. Dac[ n  N, n  2 este fixat, rezolva\i @n (0,  ) ecua\ia:


(2 x  1 ) x  (3 x  1 ) x  . . .  (n x  1 ) x  n n  1 .
1 1 1 ( )
2
Prof. GIGEL BUTH, Satu Mare
L.1100. Rezolva\i @n mul\imea numerelor @ntregi ecua\ia:
2( 3 x  8  4 x  15 )  5 x  21.
Prof. DAN SECL{MAN, Craiova
L.1101. Dac[ ABCD este un patrulater convex ]i punctele M, N, P, Q sunt mijloa-
AB 2  BC 2  CQ 2  DN 2  DM 2  AP 2
cele laturilor AB, BC, CD, DA ar[ta\i c[:  1.
CD 2  DA 2  AN 2  BP 2  BQ 2  CM 2
Prof. NECULAI STANCIU, Buz[u
L.1102. Determina\i f, g : R  R , care pentru orice x  R verific[ simultan rela\iile:
(i) x 3  f 3 (x )  3x  f (x )  g 2 (x )  cos x; (ii) g 3 (x )  3x 2  f 2 (x )  g(x )   sin x.
Prof. dr. DORIN M{RGHIDANU, Corabia, Olt

Clasa a XI-a
L.1103. Fie matricele A, B M 3 (Z ) astfel @nc`t AB  BA ]i det A  det B  0.
Demonstra\i c[ det(A 3  B 3 ) este sum[ a dou[ cuburi de numere @ntregi.
Prof. CRISTIAN MOAN|{ , Craiova
L.1104. Fie matricea A M 2 (R) ]i x  R astfel @nc`t: det(A 2  x 2  I 2 )  0.
a) Dac[ det A  x 2  0, ar[ta\i c[ are loc inegalitatea:
det(An  x n  I 2 )  det(An  xn  I 2 )  22n  Tr(A) n , () n  N .
b) Dac[ A M 2 (Q ) ]i det(A 2  2I 2 )  0, ar[ta\i c[: A 2  2I 2 .
Prof. TRAIAN IANCULESCU, Zimnicea
x  e xn ( )
L.1105. Fie (x n ) n0 un ]ir de numere reale cu x 0  0 ]i x n  n1a ,  n  N,
unde a  (1,  ) este fixat. Dac[ P este o func\ie polinomial[ oarecare, calcula\i:
lim
n
(ln(1  P(n ) )  x n ).
Prof. CRISTIAN MOAN|{ , Craiova

62 Revista de matematic[ din Craiova


L.1106. Ar[ta\i c[ exist[ f : R  R astfel @nc`t f (f (x )  x )  a  f (x ), () x  R ]i
lim f (x )  0 dac[ ]i numai dac[ a  0.
x
Prof. DAN SECL{MAN, Craiova
L.1107. Dac[ f : a, b R este o func\ie derivabil[ cu proprietatea f (a )  0, de-
monstra\i c[ exist[ c  (a, b ), astfel @nc`t: f (c )  (b  c )  f  (c ).
Prof. dr. DORIN M{RGHIDANU, Corabia, Olt

Clasa a XII-a
L.1108. Dac[ n  N este fixat ]i a 1 , a 2 , . . ., a n  Z , demonstra\i c[ exist[ indicii
i 1 , i 2 , . . ., i n  1, 2, . . ., n astfel @nc`t: a i 1  a i 2  . . .  a i n  1(mod n ).
Prof. dr. DORIN M{RGHIDANU, Corabia, Olt

L.1109. Fie f : R  R o func\ie care admite primitive ]i n  N fixat. Demonstra\i c[


pentru orice primitiv[ F a lui f exist[ c, d  R cu propriet[\ile:
(i) f (c ) sin F(c )  c 2n1 ; (ii) f (d ) cos F(d )  d 2n1 .
Prof. DAN SECL{MAN, Craiova
 

e sin x  cos x dx ]i I   e cos x  sin x dx.


2 2

L.1110. Se consider[ integralele I 1  


0 e sin x  e cos x 2
0 e
sin x  e cos x

a) Compara\i numerele I 1 ]i I 2 .
b) Determina\i partea @ntreag[ a lui I 1 .
Prof. IONEL TUDOR, C[lug[reni, Giurgiu
L.1111. a) Dac[ a  0, a  1, b  1 ]i n  N, n  3 sunt fixate, calcula\i:
b
((1  n )x  ln a )  x n3  a x
1

I n (a, b )   dx.
x n1  a x  1
1
1
b

b) Determina\i func\iile continue g : 1 , 2  R care satisfac rela\iile:


2
2
(2  x )e x 4 4 e 1 2
1 x(x 2  e x )  g(x )dx  ln e 2  4  1 g 2 (x )dx.
2 2
Prof. TRAIAN IANCULESCU, Zimnicea

L.1112. Dac[ a  0 este fixat ]i f : a, a  1 R este continu[, ar[ta\i c[ pentru


orice n  N, n  2, exist[ punctele c 1 , c 2 , . . ., c n1  (a, a  1 ), distincte dou[ c`te dou[,
a 12
f (c 1 )  f (c 2 )  . . .  f (c n1 )
a1
cu proprietatea: a f (x )dx  n  a f (x ) dx.
Prof. DAN SECL{MAN, Craiova
*
* *

Editura Cardinal 63
RUBRICA REZOLVITORILOR
CRAIOVA (jud. Dolj): Lic. Teoretic “Tudor Arghezi”, clasa a V-a, prof. DORINA
DR{CEA: Dragomir Anda (TG 4145-4150, 4160-4162), Rotaru Mircea (G 1033-1037),
Minda Andreea (G 1033-1037), clasa a VI-a, prof. DORINA DR{CEA: Modi Francisc
Mihai (G 1038-1042), Buia Viviana (G 1038-1042), Gruescu Riana (G 1038-1042), Mitran
Rebeca (G 1038-1042), Iag[ru Adina (G 1038-1042), Sandu Alexia (G 1038-1042), Belgiug
Ianis (G 1038-1042), Ionescu Andrei (G 1038-1042), clasa a VII-a, prof. DORINA
DR{CEA: B@ra Nicolae Mihai (G 1043-1047), V@lceanu Mihai (G 1043-1047), Stoican
Andra (G 1043-1047), Iovan Cristiana (G 1043-1047), Milie] Ionu\ (G 1043-1047),
Albeanu Atia (G 1043-1047), Dahche Dina (G 1043-1047). C. N. “Elena Cuza”, clasa a V-a,
prof. MIRCEA TEREUJANU: Ioni\[ Alexandru (G 1033-1037), Drag[ Erika (G
1033-1037), Bratu Marius (G 1033-1037), Tuiu Andreea (G 1033-1037), Bechet Gabriela
(G 1033-1037), P[duraru Andrei (G 1033-1037), Laz[r Florentina (G 1033-1037), Luchici
Mihai (G 1033-1037), clasa a V-a, prof. ROXANA CRISTINA VASILE: Tob[ Paula (G
1033-1037), Olariu Teodora (G 1033-1037), Ungureanu Alex (G 1033-1037), Alb[stroiu
Lavinia (G 1033-1037), Frato]ti\eanu Evelyn (G 1033-1037), clasa a VII-a, prof. MIRCEA
TEREUJANU: Cilibiu Mihai (G 1043-1047), Manafu Alina (G 1043-1047), Mitrache
Ambra (G 1043-1047), }tef[nescu Silviu (G 1043-1047), Ispas Florin (G 1043-1047),
Cruceru Alina (G 1043-1047), Nicolicescu Andreea (G 1043-1047), Vasile Rare] (G
1043-1047), Lungu Raluca (G 1043-1047), Popescu Drago] (1043-1047), Nicola
Ana-Maria (G 1043-1047), Pacionea Ana-Maria (G 1043-1047), Speril[ Andreea (G
1043-1047), C.T.I. Alimentar[, clasa a XII-a, prof. EUGENIA TEREUJANU: Sticlan Meda
(L 1068-1072), Gogo]eanu Mirabela (L 1068-1072). }c. “Traian”, clasa a II-a, @nv.
ANDREEA DU|{: Toma Maria-Daniela (II 1-25). }c. Gim. “Gh. |i\eica”, clasa a VI-a,
prof. VIRGINIA GRIGORESCU: Aranghel Mihai (G 1038-1042), St[ncescu Robert (G
1038-1042), Dumitru Mihaela (G 1038-1042), B[ndoi Alexandru (G 1038-1042), Preda
Carina (G 1038-1042), Barbu Andrei (G 1038-1042), Ciorgan Andrei (G 1038-1042),
C@rstocea Darius (G 1038-1042). C. N. “Fra\ii Buze]ti”, clasa a X-a, prof. LUCIAN
TU|ESCU: Jianu Ligia }tefania (L 1058-1062).
CUGIR (jud. Alba): }c. Gim. Nr. 3, clasa I, prof. ANDA TEC}A: B@rlea Iulia Andreea (I
1-25), Cezar Rare] Adrian (I 1-25), Clement Alexandru Florin (I 1-25), Costea Cristian (I
1-25), Du]a David Ioan (I 1-25), Grecu Iulian George (I 1-25), Icl[nzan Andrei Augustin
(I 1-25), Jeflea Darius George (I 1-25), Mara Bianca (I 1-25), Micu Sebastian (I 1-25),
Modoran Daria Anamaria (I 1-25), Molode\ Andrei Daniel (I 1-25), Zuga Daria Maria (I
1-25), clasa I, @nv. ELISABETA HADA ]i prof. @nv. Primar SABINA PRESECAN: Ple]a
Adrian Nicolae (I 1-25), Manciuc Marius (I 1-25), G[ld[u Ioana Valentina (I 1-25), Neagu
Angelina (I 1-25), Barba Daniel Eduard (I 1-25), Ivini] Andrei (I 1-25), Lasl[u Oana
Antonia (I 1-25), Dicu Sebastian (I 1-25), Mizgai Sergiu (I 1-25), Cri]an C[t[lina (I 1-25),
Guzu Andrei Alin (I 1-25), S[lcudean Alexandra (I 1-25), Mure]an Matei (I 1-25), Duca
Andrei (I 1-25), Duca Alin (I 1-25), Artene Lavinia (I 1-25), clasa a II-a, @nv. LAVINIA
FLE}ERIU: Fle]eriu Carol (II 1-25), clasa a III-a, @nv. ELENA CRI}AN: Scutaru Bianca
(III 1-25), One\ Cristian (III 1-25), Pahone Amelie (III 1-25), Herlea Alexia (III 1-25),
Buda Gelu (III 1-25), Andrasiu Bogdan (III 1-25), Simina Buble] Radu (III 1-25), Mitea
Denisa (III 1-25), Teban Sergiu (III 1-25), Todescu Ioana Alexandra (III 1-25), Dima
Maria (III 1-25).
LUPENI (jud. Hunedoara): }c. Gim. Nr. 2 - Lic. Teoretic “Mircea Eliade”, clasa a III-a,
@nv. ELENA DULEAN|U: Duduianu M. Cristina (III 1-25), Pal Adonis (III 1-25), Temelie
Anca Laura (III 1-25), clasa a V-a, prof. EMILIA VELCEA: }oac[ Bianca (TG 4148-4152,

64 Revista de matematic[ din Craiova


4160, 4162, 4165), Topor Vlad-Mihai (TG 4148-4152, 4160, 4162), Ariton Diana (TG
4148-4152, 4160, 4162), Anghel Marian Ion (TG 4148-4151, 4160), R[dulic[ Briana (TG
4148-4152, 4160, 4162), Nicolae Robert (TG 4148-4152, 4160, 41620, Bartha Vanesa (TG
4148-4152, 4160, 4162, 4165), Silaghi Monica (TG 4148-4152, 4160, 4162), Bistrian David-
Ilie (TG 4149, 4150, 4160), Boncioag[ Theodora (TG 4148-4152, 4160, 4162), clasa a VI-a,
prof. EMILIA VELCEA: Anghelu] Anca (TG 4176, 4177, 4184-4186), Manole Radu (TG
4171, 4173, 4176, 4177, 4184-4186), Drede\an Daniela (TG 4171, 4173, 4176-4178, 4184,
4185), Neam\u Laura (TG 4171, 4176-4178, 4184-4186), But[ Ariana (TG 4173, 4176-
4178, 4184-4186), Brozb[ Roxana (TG 4148, 4173, 4176, 4177, 4185, 4186), Mihai Alexan-
dra (TG 4176-4178, 4184-4186), Csiszer Karolina (TG 4176-4178, 4184-4186), }elaru
Oana (TG 4176-4178, 4184), clasa a VII-a, prof. EMILIA VELCEA: Eles Maria (TG 4195-
4197, 4220), |iparu Gabriel (TG 4195-4197, 4220), Jurca Florentina (TG 4195-4197,
4220), Barna Andrei (TG 4195-4197, 4220), C[pit[nescu Bianca (TG 4195-4197, 4220),
Sterie Darius (TG 4195-4197, 4220), Dorneanu Diana (TG 4195-4197, 4220), Lup]an
Adrian Florin (TG 4195-4197, 4220), clasa a VII-a, prof. GHEORGHI|A TRAICU:
Some]an Bogdan-Andrei (TG 4203, 4208, 4209, 4219; G 1046), Lungu Claudiu (TG 4195-
4197), clasa a VIII-a, prof. ALINA NICOLAESCU: Lu\a] Elena (TG 4202, 4207, 4208,
4221, 4225, 4226, 4228; G 1044).
SATU-MARE: C. N. "Doamna Stanca", clasa a V-a, prof. GIGEL BUTH: Masni\a
Anamaria (G 1033-1037), Karpinszki Levente (IV 1-25), Pereni Carina (G 1035, 1036; TG
4149, 4150, 4158, 4160, 4165), clasa a X-a, prof. GIGEL BUTH: Feier Denisa (L 1059, 1060,
1069, 1070), Ciorca] Cristian (G 1049), Gabor Blanka (G 1049, 1051), Belbe Ana (TL
3599, 3601, 3603, 3606-3608; G 1036), Maxim D[nu\ (TL 3588, 3591, 3593, 3600, 3603; L
1058-1060), Balogh Vivienne (L 1053, 1059), clasa a XI-a, prof. GIGEL BUTH: Ani]orac
Denisa (L 1058, 1059), Pipa] Denisa (L 1058, 1059), Dobran Denisa (L 1058, 1059),
Mure]an Florin (TL 3586, 3610, 3625, 3632, 3633), Gati Bianca (TL 3586, 3610, 3625,
3632, 3633).
TG - C{RBUNE}TI (jud. Gorj): }c. Gim. G. Usc[tescu, clasa a V-a, prof. MIRCEA
POPESCU: Pas[re Vl[du\ (G 1033, 1036), Pas[re Denis Florin (G 1033).
TG - JIU (jud. Gorj): C. N.”Spiru Haret”, clasa a XI-a, prof. GRIGORIE PREOTEASA:
Lic[ Elisa-Roxana (TL 3610, 3612, 3613, 3632), clasa a XII-a, prof. GRIGORIE
PREOTEASA: Guran Oana (TL 3613, 3638). }c. Gim. “Constantin Br`ncu]i”, clasa a V-a,
prof. SORIN MATEI TODEA: Croitoru }tefania (TG 4145, 4146, 4160), Deaconescu
Darius (TG 4145, 4146), Pl[]toi Roxana (TG 4146-4148), Ciulavu Daria (TG 4146, 4147,
4155, 4160), clasa a VI-a, prof. DANIEL STRINU: Croitoru Elena (TG 4171, 4188, 4191,
4193), Avram Elena Bianca (TG 4190-4193), clasa a VI-a, prof. SORIN MATEI TODEA:
Piscureanu Elena-}tefania (TG 4170, 4173, 4179, 4180), Bac@tea Andrei (TG 4179, 4180),
Olteanu Leonard (TG 4177, 4179), Vieru Andreea (TG 4177, 4180), Neam\u C[t[lina (TG
4170, 4173, 4177, 4180), Popescu Izabela (TG 4172, 4173, 4187), Anghel Andreea (TG
4172, 4173), G[man Cosmina (TG 4170, 4171, 4187), Dondera Anabela (TG 4170-4173),
Negreanu Alexandra (TG 4172, 4173, 4177, 4179, 4180), B[lu Elena Cornelia (TG 4177,
4179, 4180), Calot[ Delia (TG 4177, 4179, 4180), P[ulescu Nicola (TG 4177, 4179, 4180),
clasa a VII-a, prof. SORIN MATEI TODEA: Zanficu M[d[lina (TG 4195, 4196), Radu
Alexandra (TG 4196, 4204), M[r[cine }tefania (TG 4195, 4198, 4200), Madrea Elena (TG
4195, 4198), Todea Teodor (TG 4195, 4196, 4200, 4207), Stamatoiu Alexandru (TG 4195,
4196), Maghiar Alexandru (TG 4195, 4198, 4200), Popescu Eduard (TG 4195, 4198,
4200), clasa a VII-a, prof. DANIEL STRINU: Ciocioi Ionela (TG 4206), }c. Gim. “Pompiliu
Marcea”, clasa a VIII-a, prof. CONSTANTIN Z{LOG: Popescu Ariana (TG 4220, 4222,
4232, 4233; G 1043, 1046, 1049, 1051).

Tiparul executat la Imprimeria “Oltenia” Craiova


TURCINE}TI (jud. Gorj): }c. Gim. “Grigore Geam[nu”, clasa a V-a, prof. DORU AURE-
LIAN HORTOPAN: Ciobescu Andreea Denisa (G 1033, 1036), Ciobescu Gheorghe Giani (G
1033, 1036), Dijm[rescu Maria Amalia (G 1034, 1035), Hortopan Roberta (G 1033, 1037),
Iriza Ionu\ Cristian (G 1034, 1036), Popescu Raul }tefan (G 1033, 1034, 1037), T@rziu Andra
Melisa (G 1033, 1036, 1037), clasa a VI-a, prof. DORU AURELIAN HORTOPAN: Axinte
Mihai (G 1038, 1040), Gheorghiu Alexandru (G 1038, 1041), Grecu Georgiana (G 1039,
1040), Gu\[ Simina (G 1041, 1042), Hortopan Dumitru Marius (G 1038, 1039, 1042),
Popescu Iustin (G 1039, 1042), clasa a VII-a, prof. DORU AURELIAN HORTOPAN: Berca
Eduard (G 1043, 1044, 1046), Br[descu D[nu\ (G 1043, 1047), Roman Adelina (G 1044,
1045, 1046).
TIMI}OARA (jud. Timi]): Lic. Ped. “Carmen Sylva”, clasa a VI-a, prof. ANA PO}- TARU:
Hugeanu Andrada Daniela (TG 4170-4181, 4183-4194; G 1038, 1040), Florea Denisa Daria
(TG 4170-4173, 4175-4178; G 1038, 1039, 1041), Bu]oi Cristina (TG 4173, 4175-4178, 4180,
4184, 4187, 4192; G 1038, 1040-1042), Mitrache Anastasia (TG 4170, 4171, 4173, 4177, 4179,
4180, 4183, 4185-4187), clasa a VII-a, prof. ANA PO}- TARU: }oica Irina (TG 4195-4197,
4202-4204, 4207, 4209, 4214), Mohammadi Iasmina (TG 4173, 4174, 4179, 4183, 4186, 4196,
4197, 4199, 4202, 4203, 4207; G 1039, 1044), Igna Iulia (TG 4195-4197, 4202-4205, 4207,
4209; G 1036, 1039), C. Tehnic “Ion Mincu”, clasa a VI-a, prof. DELIA MITRAN: Kondora
Norbert (TG 4171, 4172).
URECHE}TI (jud. Gorj): }c. Gim. Ureche]ti, clasa I, @nv. VALENTINA-IRINA COVER-
C{: Oproiu Emilia Teodora (I 1-25), B[r[gan Vasile-Alin (I 1-25), Eremia Irinel (I 1-25),
Sanda Eugenia-Daniela (I 1-25), Balt[ Alexandru-Valentin (I 1-25), Cojocaru Petru\a
Nicoleta (I 1-25), Gu\[ Constantin-Georgian (I 1-25).
ZIMNICEA (jud. Teleorman): Lic. Teoretic Zimnicea, clasa a X-a, prof. TRAIAN IANCU-
LESCU: Ianculescu Liviu (TL 3585-3587, 3589, 3590, 3593, 3594, 3601, 3602, 3606), Heda
}tefan (TL 3584, 3589, 3600, 3602, 3606), Rotaru Ionel (TL 3584, 3587, 3589, 3600, 3602,
3606), T@n\arcu Paul-Ionu\ (TL 3584, 3587, 3589, 3600, 3602, 3606), Cocean Isabella (TL
3584, 3587, 3589, 3600, 3602, 3606), clasa a XII-a, prof. TRAIAN IANCULESCU: P@rghie
Florentina (TL 3638, 3642, 3648, 3657), C[p[\@n[ Alin (TL 3638, 3648, 3657), Fartalnicu
Diana (TL 3638, 3642, 3648, 3657), C[ld[ru][ Marilena-Roxana (TL 3638, 3642, 3648, 3657),
C@nzeac[ M[d[lina (TL 3638, 3642, 3648, 3657), Calot[ Laura (TL 3638, 3642, 3648, 3657),
Camber Denisa (TL 3594, 3638, 3642, 3648, 3657), Conu Adriana (TL 3638, 3642, 3648,
3657), Delcea Laura Elisa (TL 3638, 3642, 3648, 3657), Ciupitu Alina (TL 3638, 3642, 3648,
3657).

NOTA REDAC|IEI: TALON DE PARTICIPARE
Solu\iile problemelor
propuse se vor trimite pe Nume..........................................Prenume..................................
adresa redac\iei @nso\ite
de talonul de participare Localitatea.........................................Jud...................................
completat p`n[ la data de }coala.........................................................Clasa........................
15.07.2015.
Profesor.......................................................................................


I.S.S.N. 1221- 3551

Lei 8

S-ar putea să vă placă și